You are on page 1of 113

AP

__________________________
Sample AP World History
2017 Multiple Choice Questions
__________________________
WORLD HISTORY

SECTION I, Part A

Time—55 minutes

55 Questions

Directions: Each of the questions or incomplete statements below is followed by four suggested answers or
completions. Select the one that is best in each case and then fill in the appropriate letter in the corresponding space
on the answer sheet.

Questions 1 - 3 refer to the excerpt below.

“The first object that saluted my eyes when I arrived on the coast was the sea, and a slave ship…waiting for its
cargo. These filled me with astonishment, that was soon converted into terror…I was immediately handled and
tossed up to see if I was sound, by some of the crew; and I was now persuaded that I had got into a world of bad
spirits, and that they were going to kill me… I was soon put down under the decks, and there I received such a
salutation in my nostrils as I had never experienced in my life: so that, with the loathsomeness of the stench, and
with my crying together, I became so sick and low that I was not able to eat…I now wished for the last friend, death,
to relieve me; but soon, to my grief, two of the white men offered me eatables; and on my refusing to eat, one of
them held me…and laid me across, I think, the windlass, and tied my feet, while the other flogged me severely...”

Elaudah Equiano, The Interesting Narrative of the Life of Elaudah Equiano. 1789

1) Equiano's account suggests that cultural


encounters between Europeans and Africans
in the 1700's was most directly shaped 2) Equiano's account contributed most
by directly to which of the following trends?

(A) Muslim influences (A) The growth of the ideology of Social


Darwinism
(B) interest in science and technology
(B) greater political autonomy for Africa
(C) Trans-Atlantic Exchanges
(C) demands for worker's rights
(D) reliance on agriculture
(D) the abolition of international slavery

3) The experience described in the excerpt


is an example of which of the
following historical developments?

(A) The Industrial Revolution

(B) The Triangle Trade

(C) The development of Conservativism

(D) The influence of Nationalism


Questions 4 - 7 refer to the graph below.

GROWTH OF THE WORLD POPULATION, 500 B.C.- 2025 C.E

WordPress.com

4. Which of the following was a


significant cause of the trend from 1800
to 1900 shown in the graph? 5. The people represented by the
graph from 1999-2025 most typically
(A) Active encouragement of will live in which of the following
urbanization by the Muslim regions of the world
merchants
(A) Asia
(B) Economic and political difficulties in
Africa & Asia (B) Africa

(C) Incentives offered by European (C) The Americas


companies looking to hire foreign
(D) Europe
migrants from colonial territories

(D) The urbanization of Europe during


the Industrial Revolution
7. The main trend shown in the graph
was most directly associated with which
6. Which of the following was a direct of the following processes occurring in
effect of the trend in population growth Europe at the time?
between 1800-1900 shown on the
graph? (A) The divergence of European and
colonial economies
(A) An increase in tensions between
social classes (B) The beginning of an industrialized
economy
(B) A major economic depression
(C) The migration of Asians into Europe
(C) An upsurge in religious sentiment
(D) The revival of Catholicism
(D) The development of political
elections
Questions 8 - 10 refer to the excerpt below.

Preamble
"Mothers, daughters, sisters [and] representatives of the nation demand to be constituted into a
national assembly. Believing that ignorance, omission, or scorn for the rights of woman are the only
causes of public misfortunes and of the corruption of governments, [the women] have resolved to set forth
a solemn declaration the natural, inalienable, and sacred rights of woman in order that this declaration,
constantly exposed before all members of the society, will ceaselessly remind them of their rights and
duties..."

Article I
"Woman is born free and lives equal to man in her rights. Social distinctions can be based only on
the common utility."

Olympe de Gouges, Declaration of the Rights of Woman and Citizen. 1791

8. The sentiments conveyed in the 9. Which of the following developments


excerpt most directly opposed the in the first half of the twentieth century
prevailing ideal in the early best represented the continuation of
eighteenth century that the ideas expressed in the
declaration?
(A) the ability of women to earn wages
was a positive development (A) The 19th Amendment to the U.S.
Constitution, granting women the right
(B) women should focus on the home to vote
and the domestic sphere
(B) Women’s support for WWI
(C) women shouldn't enjoy full and
equal rights with men (C) Support for outlawing the
production and sale of alcohol
(D) women should educate their
children about the rights and (D) A movement focused on religious
responsibilities of citizenship revivals and personal conversion

10. Many female supporters of the


French Revolution in 1789 broke ranks
with which group by 1800's?

(A) The Conservatives

(B) Radical supporters of the French


Revolution

(C) Supporters of Napoleon


(D) The Catholic Church

Questions 11 - 13 refer to the excerpts below.

"... Stories have been related to me, which the hearer can scarcely credit, as to the terror of the Tatars (Mongols),
which God Almighty cast into men's hearts; so that it is said that a single one of them would enter a village or a
quarter wherein were many people, and would continue to slay them one after another, none daring to stretch forth
his hand against this horseman. And I have heard that one of them took a man captive, but had not with him any
weapon wherewith to kill him; and he said to his prisoner, "Lay your head on the ground and do not move," and he
did so, and the Tatar went and fetched his sword and slew him therewith.
Ibn al-Atir, 1220-1221, Muslim historian

“Hulagu Khan handed the rich treasures which had been brought...from the Caliph’s court to the ruler of Rayy…for
safekeeping, and had them carried to Azerbaijan as was the booty from Asia Minor, Georgia, Armenia, Luristan and
the land of the Kurds…..These invaders burned our great libraries, broke our canals and ditches, destroyed our
farms, defiled the true Faith by raising temples to Buddha…attempted to destroy our trade with paper money…”
Muslim historian, Rashid Fadl Abi-l’Hair, 1498

13. The ideas expressed by both Muslim


historians best account for which of
11. The excerpts were written in the following features of the Silk
response to the Road during and immediately after the
Mongol conquest of Baghdad.
(A) Mongol's policy of promoting Islam
(A) The increase in tension between
(B) Arab empires' failure to protect
nomads and settled populations
citizens from the Mongol invasions
(B) The rapid expansion of Islam from
(C) Arab Dynasties failures to
Spain to India
promote cross-cultural trade
(C) The inability of any empire to
(D) The Mongols ability to conquer
successfully resist Mongol invasions
previously wealthy and powerful
empires (D) The spread of disease and the
eventual collapse of the Silk
12. The ideas about invasion and
Road
conquest expressed by both Muslim
historians are most consistent with
which of the following?

(A) The military and political traditions


of central Asian nomads

(B) The belief in Islam

(C) The concept of religious


intolerance

(D) The ideas of the Middle Ages


Questions 14 - 17 refer to the excerpts below.

“His Majesty's government view with favour the establishment in Palestine of a national home for the Jewish
people, and will use their best endeavours to facilitate the achievement of this object, it being clearly understood that
nothing shall be done which may prejudice the civil and religious rights of existing non-Jewish communities in
Palestine, or the rights and political status enjoyed by Jews in any other country."
British Foreign Secretary John Balfour, Balfour Declaration, 1917

14. Which of the following was the 16. Which of the following sets of
most immediate result of the United Nations decisions demonstrated
declaration? the strongest continuity with the idea
expressed in the excerpt?
(A) Arab nationalist became
increasingly united over in their political (A) U.N. Israel/Palestine Partition Plan
strategies of 1947

(B) the rise in concern about the (B) Camp David Agreement
persecution of Jews in Europe
(C) Oslo Accords
(C) the rise of Zionism promoting
Israel as a homeland for the Jews (D) UN Security Council Resolution 242,
requiring Israel to withdraw from any
(D) The British refused to support the recently occupied territories
idea and focused on Arab nationalism

15. The decision excerpted most 17. The Balfour Declaration stood in
directly reflected a growing belief after contrast mostly to which of the
the First World War that the world's following other decisions?
political borders should be divided
according to (A) Sykes-Picot Agreement, granting
Britain control of Mandate Palestine
(A) racial boundaries
(B) The creation of the state of Israel
(B) social class
(C) The withdraw of Israeli forces from
(C) national boundaries the Sinai Peninsula

(D) fully autonomous and democratic (D) The UN Partition Plan of 1947
lines
Questions 18 - 20 refer to the excerpt below.

"When the Spanish first journeyed there, the indigenous population of the island of Hispaniola stood at
some three million; today only two hundred survive. The island of Cuba... is now to all intents and purposes
uninhabited;" and two other large, beautiful and fertile islands, Puerto Rico and Jamaica, have been similarly
devastated. Not a living soul remains today on any of the islands of the Bahamas, which lie to the north of
Hispaniola and Cuba, even though every single one of the sixty or so islands in the group, as well as those known as
the Isles of Giants and others in the area, both large and small, is more fertile and more beautiful than the Royal
Gardens in Seville and the climate is as healthy as anywhere on earth. The native population, which once numbered
some five hundred thousand, was wiped out by forcible expatriation to the island of Hispaniola, a policy adopted by
the Spaniards in an endeavour to make up losses among the indigenous population of that island."

Bartolome de las Casas, A Short Account of the Destruction of the West Indies, 1542

18. The ideas expressed in the excerpt


differed from the predominant Spanish
approach to colonization primarily
because de las Casas was
19. The excerpt best reflects an effort
(A) encouraging Spain to start by De Las Casas to
colonizing the Caribbean
(A) overcome opposition to
(B) highlighting the decline of the colonization
natives as part of colonization
(B) persuade other priests to join him
(C) promoting new laws that would in the Americas
make it easier to control the natives
(C) encourage the improved treatment
(D) arguing that the Americas is more of the natives
fertile than Europe, making it suitable
(D) prevent the abuses of colonization
for colonization
from spreading to the Caribbean

20. Which of the following best


represents continuity in the years after
1800 with the account that De Las Casas
described in the excerpt?

(A) the treatment of slaves as part of


the Trans-Atlantic Slave trade
(B) Latin American independence (D) Simon Bolivar's struggle to unify
movements South America into a gran Colombia

(C) the violence and bloodshed during


the Cuban Revolution

Questions 21 - 23 refer to the excerpts below.

"After China has established a powerful government, we must not be afraid, as Western people are, that the
government will become too strong and that we will be unable to control it. For it is our plan that the political power
of the reconstructed state be divided into two parts. One is the power over the government; that great power will be
placed entirely in the hands of the people, who will have a full degree of sovereignty and will be able to control
directly the affairs of the state."
Sun Yet Sen, The Principle of Democracy, 1924

"Yet if the Western world is still determined to rule mankind by force, then Africans, as a last resort, may have to
appeal to (use) force in the effort to achieve freedom . . . We are determined to be free. We want education. We want
the right to earn a decent living, the right to express our thoughts and emotions, to adopt and create forms of beauty.
We demand for Black Africa autonomy (self-rule) and independence"
Jomo Kenyatta, Challenge to Colonial Powers, 1945

21. The statements of both Sun Yet Sen


and Jomo Kenyatta share the same goal
of?

(A) using violence as a means of


achieving freedom

(B) breaking free from the chains of


22. Sun Yet Sen issued the speech
colonization to establish an
primarily in order to
independent nation
(A) show the state along with the
(C) spreading the ideals of democracy
people will influence political affairs
across the entire region
(B) gain support from the Communists
(D) avoiding a military conflict with the
imperialists (C) protect nationalists interests in
Manchuoko

(D) support the landowning classes of


Chinese society

23. Kenyatta's speech best reflects


which of the following developments
in the decolonization of Africa?
(A) the expansion of political violence
and civil wars across Africa

(B) the expansion of Pan-Africanism

(C) the Scramble for Africa

(D) the demand of colonial nations for


independence and decolonization

Questions 24 - 26 refer to the late 20th century photograph by Stuart Franklin.

© Stuart Franklin/Magnium Photos (TheGuarian.com)

(D) an increase in media attention for


civil disobedience
24. Conditions like those shown in the
image contributed most directly to
which of the following?

(A) an increase in Socialist sentiments

(B) the passage of democratic reforms


under Communism and Dictatorship 25. The conditions shown in the image
depict which of the following trends in
(C) a decline in the use of violence the late 20th century?
against citizens
(A) the isolation of the urban working
class

(B) China's growing military

(C) the corruption of Communist


governments

(D) the growing gap between


Communist government and its people

26. Advocates for individuals such as


those shown in the image would have
most likely agreed with which of the
following perspectives?

(A) a strong economy should be well


regulated to benefit all

(B) the people should be represented


in the military

(C) the government should be


responsive to the needs of the people

(D) Laissez-Faire style capitalism will


lead to economic benefits for workers
Questions 27 - 29 refer to the excerpt below.

"That the various forms of epidemic, endemic, and other disease...chiefly amongst the labouring
classes...produced by decomposing animal and vegetable substances, by damp and filth, and close and overcrowded
dwellings prevail amongst the population in every part of the kingdom, whether dwelling in separate houses, in rural
villages, in small towns, in the larger towns...Contaminated London drinking water containing various micro-
organisms, refuse (trash), and the like. The high prosperity in respect to employment and wages, and various and
abundant food, have afforded to the labouring classes no exemptions from attacks of epidemic disease...That the
formation of all habits of cleanliness is obstructed by defective supplies of water.That the annual loss of life from
filth and bad ventilation are greater than the loss from death or wounds in any wars in which the country has been
engaged in modern times.

Sir Edwin Chadwick, Inquiry Into Sanitary Conditions of the Working Class of England, 1842

27. Which of the following aspects of


Chadwick's description expresses a
major change in Europeans' views of
sanitation?

(A) London water has been


28. Chadwick's ideas are most directly a
contaminated
reaction to the
(B) religion has a role to play in
(A) the increase in social tensions and
ensuring sanitation codes
abuse of industrial workers
(C) every worker should be responsible
(B) growing middle class of urban
for his/ her own hygiene
workers
(D) the government should ensure
(C) the increase in demand for raw
basic sanitation standards for workers
materials for the new factories

(D) the mechanics of the factory


system

29. Chadwick's position regarding the


sanitary conditions of the workers was
most strongly supported by which of (B) Factory owners
the following?
(C) Members of Parliament
(A) Anti-imperialists
(D) Communists

Questions 30-33 refer to the excerpt below.

"Reading this proclamation, I have concluded that the Westerners are petty indeed. It is impossible to reason with
them because they do not understand larger issues as we understand them in China. There is not a single Westerner
versed in Chinese works, and their remarks are often incredible and ridiculous. To judge from this proclamation,
their religion is no different from other small, bigoted sects of Buddhism or Taoism. I have never seen a document
which contains so much nonsense. From now on, Westerners should not be allowed to preach in China, to avoid
further trouble."
Qing Emperor Kangxi, Decree on Christianity, 1721 

30. The decree imposed by Kangxi was a


response to the?

(A) controversy regarding support for


various minority religions

(B) debate over the legality of


Confucianism within the Qing empire
31. The ideas expressed in Kangxi's
(C) debate over the treatment of decree most strongly influenced which
Christian missionaries in East Asia event during the Qing era?

(D) the dispute over monotheism (A) The Taiping Rebellion

(B) the fall of Empress Dowager Xi

(C) the Opium Wars

(D) The Boxer Rebellion

32. Which of the following groups most


strongly opposed Kangxi's point of
view in the decree?

(A) The Chinese peasants and farmers


(B) the Confucian scholar-gentry

(C) Christian missionaries

(D) European merchants

33. Which of the following


developments demonstrated the
strongest continuity with the idea
expressed in the decree?

(A) The increase in social tensions


between the government and peasants

(B) The development of Christianity as


a minority in China

(C) The decline of the scholar gentry


class

(D) The spread of Christianity


throughout parts of southern China
Questions 34 - 36 refer to the excerpts below.

The Columbian Exchange, is one of the more spectacular and significant ecological events of the past millennium.
When Europeans first touched the shores of the Americas, Old World crops such as wheat, barley, rice, and turnips
had not traveled west across the Atlantic, and New World crops such as maize, white potatoes, sweet potatoes, and
manioc had not traveled east to Europe. In the Americas, there were no horses, cattle, sheep, or goats, all animals of
Old World origin. Except for the llama, alpaca, dog, a few fowl, and guinea pig, the New World had no equivalents
to the domesticated animals associated with the Old World, nor did it have the pathogens associated with the Old
World’s dense populations of humans...Among these germs were those that carried smallpox, measles, chickenpox,
influenza, malaria, and yellow fever.

Alfred Crosby, The Columbian Exchange, 1978

34. The import of Old World crops to


the New World transformed American
society mostly by?

(A) diversifying diets and thereby


triggering a population increase

(B) encouraging the privatization of


35. The patterns described in the
lands
excerpt most directly foreshadowed
(C) improving relations between which of the following developments?
Europeans and native inhabitants
(A) the spread of corn from South
(D) encouraging warfare between America to North America
European powers
(B) the gradual transition from a feudal
to capitalist society

(C) The population decline of Native


Americans

(D) the emergence of racially mixed


populations in the Americas
36. The trends described by Crosby
most closely corresponds with which of
the following major historical
developments in the Atlantic world?

(A) The rise of the Triangle Trade

(B) The Manila Galleons

(C) The growth of imperialism in Africa

(D) The Plague


Questions 37 - 39 refer to the map below.
The Silk Road Trade Routes around 600 C.E.

37. The pattern of trade routes after


600 C.E. were linked most directly by
which of the following factors?

(A) Abassid Empire's attempts to


impose taxes on non-Muslims

(B) the fall of the western Roman


38. The difference in trade route
empire
patterns from the land to sea had which
(C) the role of the Monsoon winds of the following effects?

(D) The rising influence of Muslim (A) promoted the advancement and
governments and merchants diffusion of maritime technology
(B) a decrease in the significance of the
monsoon winds

(C) an increase in conflict between the


Chinese and Arab merchants

(D) an increase in trade with western


Europe

39. The difference in trade patterns


between the land and sea routes best
explains the

(A) significance of understanding how


to navigate the Monsoon winds

(B) development of religious


differences between Africans, and
Persians

(C) spread of the Bubonic Plague

(D) growth of religious tensions


between Christians and Muslims
Questions 40 - 42 refer to the excerpt below.

"... No one ought to harm another in his life, health, liberty, or possessions... ; for without this the law could
not have that which is absolutely necessary to its being a law, the consent of the society, over whom nobody can
have a power to make laws but by their own consent and by authority received from them….: They must not raise
taxes on the property of the people without the consent of the people.... When any one...make laws without
authority, which the people are not therefore bound to obey;...and may constitute to themselves a new legislative..."

John Locke, Two Treatises on Government, 1689

40. Which of the following groups


would have been most likely to support
Locke's views expressed in the excerpt?

(A) Conservative landowners

(B) liberal revolutionaries 42. In the 18h century, the views


expressed by Locke most directly
(C) Absolute monarchs contributed to

(D) members of the clergy (A) increased social tension between


landlords and peasants

(B) the development of Feudalism in


41. Which of the following most directly Russia
undermines Locke's claims?
(C) the rise of social divisions within
(A) king's rarely give up their power European society
(B) the development of the Trans- (D) the drafting of the American
Atlantic slave network Declaration of Independence
(C) many colonists engaged in forms of
resistance to taxation

(D) a majority of Enlightenment


philosophes were deists
Questions 43 - 44 refer to the excerpt below.

"Now the Persian nation is made up of many tribes. Those which Cyrus assembled and persuaded to revolt
from the Medes were the principal ones on which all the others are dependent. These are the Pasargadae, the
Maraphians, and the Maspians, of whom the Pasargadae are the noblest. The Achaemenidae, from which spring all
the Perseid kings, is one of their clans... The customs which I know the Persians to observe are the following: they
have no images of the gods, no temples nor altars, and consider the use of them a sign of folly. This comes, I think,
from their not believing the gods to have the same nature with men, as the Greeks imagine. Their wont, however, is
to ascend the summits of the loftiest mountains, and there to offer sacrifice to Zeus, which is the name they give to
the whole circuit of the firmament. They likewise offer to the sun and moon, to the earth, to fire, to water, and to the
winds. These are the only gods whose worship has come down to them from ancient times."

Herodotus, On the Customs of the Persians, 430 B.C.E.

43. The excerpt would be most useful to


historians as a source of information
about which of the following?

(A) the similarities and difference of 44. Which of the following was a major
Greek and Persian gods contrast between the customs of the
Persians and those of the Greeks?
(B) the nature of ruler and citizen in
ancient Persia (A) the Persians have no images of
gods nor altars for religious worship
(C) the interaction between Greeks and
Persians in the classical age (B) Greeks had less gods than Persians

(D) the role that religion has played (C) the Greek economy was based on
historically in Persian culture agriculture

(D) the Persians offered sacrifice to


their gods
Questions 45 - 47 refer to the excerpt below.

" When Motecuhzoma [Montezuma] had given necklaces to each one, Cortés asked him: "Are you Motecuhzoma?
Are you the king? Is it true that you are the king Motecuhzoma?"And the king said: "Yes, I am Motecuhzoma."
Then he stood up to welcome Cortés; he came forward, bowed his head low and addressed him...

The Spaniards attacked the musicians first, slashing at their hands and faces until they had killed all of them. The
singers-and even the spectators- were also killed. This slaughter in the Sacred Patio went on for three hours. Then
the Spaniards burst into the rooms of the temple to kill the others: those who were carrying water, or bringing fodder
for the horses, or grinding meal, or sweeping, or standing watch over this work. The Sun had treacherously
murdered our people on the twentieth day after the captain left for the coast. We allowed the Captain to return to the
city in peace. But on the following day we attacked him with all our might, and that was the beginning of the war."

An Aztec Account of the Conquest of Mexico, c. 1520's

45. The Aztec account most directly


illustrated the debates about which of
the following issues in America?

(A) The process of breaking colonial


laws

(B) The issuing of the "Requirement"


46. The actions of the Spanish
(C) The colonization of the Caribbean described in the Aztec account
contributed to which of the following
(D) The treatment of "the Other" upon economic developments within the first
first contact in the New World century of colonization in the
Americas ?

(A) the establishment of the


encomienda system

(B) the conquest of the Maya by Cortez

(C) the rise of anti colonial governors


caused by the mistreatment of natives
(D) the reliance on the Trans-Atlantic
Slave Trade for labor

47. Efforts by the Aztecs to defeat the


Spanish militarily ultimately failed
because

(A) the more powerful tribes in the


area allied with Spain against the Aztecs

(B) the Aztecs were divided among


various Mexican tribes

(C) Aztecs had no immunities to


disease while the Europeans had
superior metallurgy and technology

(D) They hesitated to attack the


Spanish when they had the chance
Questions 48 - 50 refer to the excerpt below.

“The most celebrated system of jurisprudence known to the world begins, as it ends, with a Code. From the
commencement to the close of its history, the expositors of Roman Law consistently employed language which
implied that the body of their system rested on the Twelve Decemviral Tables, and therefore on a basis of
written law. Except in one particular, no institutions anterior to the Twelve Tables were recognised at Rome.
The theoretical descent of Roman jurisprudence from a code, the theoretical ascription of English law to
immemorial unwritten tradition, were the chief reasons why the development of their system differed from the
development of ours. Neither theory corresponded exactly with the facts, but each produced consequences of
the utmost importance.
Henry Maine, Lawyer and Historian, Ancient Law, 1861

48. The Twelve Tables were significant


in the history of law because they

(A) were the first successful attempt


of the Romans to codify law

(B) were the first patriarchal laws

(C) discriminated against slaves, 49. The Twelve Tables most directly
women, and landless peasants reflected?

(D) promoted the study of law among (A) conflicts from the imperial Roman
the Plebian class of society period

(B) the political problems caused by


invading groups from Europe

(C) social tensions between the


Patrician and Plebian classes

(D) the conflict between urban and


rural populations

50. Which of the following evidence


would best support Maine's argument
in the excerpt?
(A) Political documents in modern law (C) Narratives from the lives of Roman
codes that reflect the principles of the politicians who drafted the Twelve
Twelve Tables Tables

(B) Testimony from government (D) Statistical data revealing the


officials that the Twelve Tables was number of Plebians and Patricians tried
influential on modern politics as a result of the Twelve Tables

Questions 51 - 53 refer to the political cartoon below.

Courtesy of Wikimedia Commons (D) promote women's equality during


the era of high imperialism
51. The political cartoon was intended
to

(A) advocate for the Chinese to stand


up to the imperialists

(B) critique how imperialism operated


in China

(C) show how the Europeans were 52. The poster most directly reflects
divided along political lines the
(A) emergence of Japan as an imperial
power

(B) the strength of the German and


French militaries

(C) the violence caused by cross-


cultural trade

(D) inability of China to resist the


imperialists

53. Which of the following represents a


later example of the actions
demonstrated in the cartoon ?

(A) The Japanese invasion of


Manchukuo in the 1930's

(B) The Chinese Civil war between the


Nationalists and Communists from 1911
to 1945

(C) The Chinese Communist Revolution


in 1945

(D) the growing tide of nationalism and


decolonization in Asia
Questions 54 - 55 refer to the excerpt below.

"Good evening. Today, our fellow citizens, our way of life, our very freedom came under attack in a series
of deliberate and deadly terrorist acts. The victims were in airplanes, or in their offices; secretaries, businessmen and
women, military and federal workers; moms and dads, friends and neighbors. Thousands of lives were suddenly
ended by evil, despicable acts of terror... America was targeted for attack because we’re the brightest beacon for
freedom and opportunity in the world. And no one will keep that light from shining... The search is underway for
those who are behind these evil acts. I’ve directed the full resources of our intelligence and law enforcement
communities to find those responsible and to bring them to justice. We will make no distinction between the
terrorists who committed these acts and those who harbor them... America and our friends and allies join with all
those who want peace and security in the world, and we stand together to win the war against terrorism."

Former U.S. President George W. Bush, Address to the Nation, 9/11/2001

(C) the U.S. invasions of Afghanistan


and Iraq

54. Which of the following actions by


the Bush Administration best reflects
the ideas about winning the war against
terrorism expressed in the excerpt ? 55. The ideas expressed by Bush in the
excerpt were most similar to those of
(A) the establishment of an American which twentieth century world leader?
command center in Africa
(A) Ho Chi Minh
(B) the decision to give U.S. foreign aid
to nations that support American (B) Franklin Delano Roosevelt
interests
(C) Woodrow Wilson

(D) Joseph Stalin

(D) the attempted overthrow of the


Venezuelan President, Hugo Chavez
END OF PART A
IF YOU FINISH BEFORE TIME IS CALLED, YOU MAY
CHECK YOUR WORK ON PART A. DO NOT GO ON TO
PART B UNTIL YOU ARE TOLD TO DO SO.
AP
__________________________
Sample AP World History
2017 Free-Response Questions
__________________________
Directions: Read each passage carefully. Answer on a separate paper using
complete sentences- bullet points are not acceptable. 4 questions; 50 minutes.

1) Answer a, b, and c.

a) Briefly explain ONE important similarity between Egyptian and


Mesopotamian religion in the ancient period prior to 500 B.C.

b) Briefly explain ONE important difference between Egyptian and


Mesopotamian religion in the ancient period prior to 500 B.C.

c)Briefly explain ONE historical factor that accounts for the difference that you
indicated in b.
Use the map below to answer all parts of the question that follows.

Source: http://sherylsbuddism.weebly.com/spread-of-buddhism.html

2) Using the map, answer a, b, & c.


a) Briefly explain how ONE major historical factor contributed to the
information depicted on the map.

b) Briefly explain ONE specific historical effect that resulted from the
information depicted on the graph.

c) Explain ONE way in which Asian society or politics was affected from the
information depicted on the map.
Directions: Read each passage carefully. Answer on a separate paper using
complete sentences- bullet points are not acceptable. 4 questions; 50 minutes.

Use the chart below to answer all parts of the question that follows.

Source: Independent research on European Dark Ages

1) BASED ON THE GRAPH


a) Briefly explain how ONE major historical factor contributed to the change
depicted on the graph.

b) Briefly explain ONE specific historical effect that resulted from the change
depicted on the graph.
c) Briefly explain how ONE person, event, or development from the period 500-
1500 that is not explicitly mentioned on the graph could be used to support the
information shown on the graph.

Use the passages below to answer all parts of the question that follows.

" One method of delivery alone remains to us; which is simply this...facts... But not only is a
greater abundance of experiments to be sought for and procured, and that too of a different
kind from those hitherto tried; an entirely different method, order, and process for carrying on
and advancing experience must also be introduced... For first of all we must prepare a Natural
and Experimental History, sufficient and good; and this is the foundation of all; for we are not
to imagine or suppose, but to discover, what nature does or may be made to do... Therefore in
the third place we must use Induction, true and legitimate induction, which is the very key of
interpretation. [* induction = starts with specific facts to draw more general conclusions.]

Sir Francis Bacon: First Book of Aphorisms (1620).

"Therefore . . . , invoking the most holy name of our Lord Jesus Christ and of His Most Glorious
Mother Mary, We pronounce this Our final sentence: We pronounce, judge, and declare, that
you, the said Galileo . . . have rendered yourself vehemently suspected by this Holy Office of
heresy, that is, of having believed and held the doctrine (which is false and contrary to the Holy
and Divine Scriptures) that the sun is the center of the world, and that it does not move from
east to west, and that the earth does move, and is not the center of the world..."

Documents in the Case of Galileo: Indictment, of 1633.

4. a) Describe ONE important difference between the views of science and


society expressed in the two passages.

b) For EACH of the passages, identify and explain ONE factor (such as a historical
development, an intellectual or philosophical trend, or a religious belief) that
likely informed the view of science and society expressed in the passage.

c) Identify ONE specific example of scientific discovery from 1450 to 1750 and
briefly explain how the example was supported or opposed by the view of either
document.
END OF SECTION 1

END OF SECTION 1

IF YOU FINISH BEFORE TIME IS CALLED, YOU MAY


CHECK YOUR WORK ON PART A. DO NOT GO ON TO
PART B UNTIL YOU ARE TOLD TO DO SO.
WORLD HISTORY

SECTION II

Total Time-- 1 hour, 30 minutes

Question 1 (Document-Based Question)

Suggested reading and writing time: 55 minutes

It is suggested that you spend 15 minutes reading the documents and 40 minutes writing your
response.

Note: You may begin writing your response before the reading period is over.

Directions: Question 1 is based on the accompanying documents. The documents have been edited for
the purpose of this exercise.

In your response you should do the following:

Thesis: Present a thesis that makes a historically defensible claim and responds to all parts of the
question. The thesis must consist of one or more sentences located in one place, either in the introduction
or the conclusion.

Argument Development: Develop and support a cohesive argument that recognizes and accounts for
historical complexity illustrating relationships among historical evidence such as contradiction,
collaboration, and/or qualification.

Use of the Documents: Explain the significance of the author's point of view, author's purpose, historical
context, and/or audience for at least four documents.

Contextualization: Situate the argument by explaining the broader historical events, developments, or
processes immediately relevant to the question.

Outside Evidence: Provide an example or additional piece of specific evidence beyond those found in
the documents to support or qualify the document.

Synthesis: Extend the argument by explaining the connections between the argument and ONE of the
following.

 A development in a different historical period, situation, era, or geographical area.


 A course theme and/ or approach to history that is not the focus of the essay (such as political,
economic, social, or intellectual history)
 A different discipline or field of inquiry (such as economics, government and politics, art history,
or anthropology).

1. Explain the causes of the rise of the Indian National Congress during the period 1857-1947.

Document 1

Source: A photo of Indian National Congress leaders, Jawaharlal Nehru and Mohandas Gandhi
Document 2

MARCH 11, 1930 

MY DEAR JAWAHARLAL,
  It is nearing 10 p.m. now. The air is thick with the rumour that I shall be arrested
during the night. I have not wired to you especially because the correspondents submit
their messages for approval and everybody is working at top speed. There was nothing
special to wire about.

  Things are developing extraordinarily well. Offers of volunteers are pouring in. The
column will proceed with the march even though I may be arrested. If I am not, you
may expect wires from me, otherwise I am leaving instructions.

  I do not know that I have anything in particular to say. I have written enough. I gave
a final message this evening to a vast crowd that gathered for prayer on the sands.

  May God keep you and give you strength to bear the burden.

With love to you all,


Bapu (Gandhi)

- Letter from Gandhi to Nehru


Document 3

...We have a stronger weapon, a political weapon, in boycott. We have perceived one fact, that
the whole of this administration, which is carried on by a handful of Englishmen, is carried on
with our assistance. We are all in subordinate service. This whole government is carried on with
our assistance and they try to keep us in ignorance of our power of cooperation between
ourselves by which that which is in our own hands at present can be claimed by us and
administered by us.
The point is to have the entire control in our hands. I want to have the key of my house,
and not merely one stranger turned out of it. Self-government is our goal; we want a control over
our administrative machinery. We don't want to become clerks and remain [clerks]. At present,
we are clerks and willing instruments of our own oppression in the hands of' an alien
government, and that government is ruling over us not by its innate strength but by keeping us
in ignorance and blindness to the perception of this fact. Professor Seeley shares this view.
Every Englishman knows that they are a mere handful in this country and it is the business of
every one of them to befool you in believing that you are weak and they are strong. This is
politics. We have been deceived by such policy so long.
What the new party wants you to do is to realize the fact that your future rests entirely in
your own hands. If you mean to be free, you can be free; if you do not mean to be free, you will
fall and be for ever fallen. So many of you need not like arms; but if you have not the power of
active resistance, have you not the power of self-denial and self-abstinence in such a way as
not to assist this foreign government to rule over you? This is boycott and this is what is meant
when we say, boycott is a political weapon. We shall not give them assistance to collect
revenue and keep peace. We shall not assist them in fighting beyond the frontiers or outside
India with Indian blood and money. We shall not assist them in carrying on the administration of
justice. We shall have our own courts, and when time comes we shall not pay taxes. Can you
do that by your united efforts? If you can, you are free from tomorrow.
- Indian politician's address to Indian National Congress (1907)
Document 4

DEHRA DUN JAIL, JULY 11, 1932

TO: THE SUPERINTENDENT, DISTRICT JAIL, DEHRA DUN

DEAR SIR,

  You were good enough to show me today the reply of the Officiating Inspector General of Prisons to my
letter dated the 22nd June. I am informed therein that, in the course of an interview with Mr. R.S. Pandit
in the Allahabad District Jail on May 27th, my wife handed a letter to Mr. Pandit, and the Jailer not
allowing this, my mother "used insulting language to the Jailer and was impertinent."

...Apart from the insult to Mr. Pandit, the Jailer's behavior was an affront to my mother and wife. My
mother hardly spoke to him.

  Three days later, on May 30th, I had my usual fortnightly interview with my mother, wife and daughter
in the Bareilly District Jail. I was then informed of what had happened. I was surprised to learn that
anyone should have behaved so discourteously to my mother and I expected some expression of regret
from the Jail officials for what had occurred. Instead of that, I now find that the Government have chosen
to punish my mother and wife. I presume this has been done on some statement made to them by the
Jailer. No reference was made, so far as I am aware, to my mother or my wife to find out what had
happened. Without any further enquiry or effort to find out the truth, the Government have not hesitated
to insult my mother and wife, and have done so in such a way as to cause the maximum inconvenience to
all parties concerned.

  It may be that it is an offence under the jail regulations to show a school report about one's children. If
Government wish to treat even this as worthy of punishment, I have no grievance. Nor shall I object if my
interviews are stopped for a month or a year. I have not come to prison for the sake of my health or for
pleasure.

  But there are certain matters which I cannot pass in silence. I cannot tolerate even the suspicion of an
affront or insult to my mother. I have noticed with deep regret that Government have not shown my
mother the courtesy which I would have expected from them under any circumstances. For the Inspector-
General to say that my mother "used insulting language to the Jailer and was impertinent" shows that he is
strangely lacking in a sense of proportion and knows little of Indian society.

  On no account am I prepared to take the slightest risk of further insult to my mother and wife. Under the
circumstances, the only course open to me is not to have any interviews, so long as I do not feel that such
interviews can be had with dignity and with no fear of discourtesy to those who come to see me. I am
therefore informing my people not to take the trouble to come for interviews with me in future, even after
the month of punishment is over...

  

Yours faithfully,

Jawaharlal Nehru

Document 5
Source: A newspaper article about the Amritsar Massacre published on April 19, 1919.

Document 6
We have now before us the data for understanding, at least in a measure, the meaning
of the "New National Movement in India." It is the awakening and the protest of a
subject people. It is the effort of a nation, once illustrious, and still conscious of its
inherent superiority, to rise from the dust, to stand once more on its feet, to shake off
fetters which have become unendurable. It is the effort of the Indian people to get for
themselves again a country which shall be in some true sense their own, instead of
remaining, as for a century and a half it has been, a mere preserve of a foreign power,
—in John Stuart Mill's words, England's "cattle farm." The people of India want the
freedom which is their right,—freedom to shape their own institutions, their own
industries, their own national life. This does not necessarily mean separation from Great
Britain; but it does mean, if retaining a connection with the British Empire,
becoming citizens,and not remaining forever helpless subjects in the hands of
irresponsible masters. It does mean a demand that India shall be given a place in the
Empire essentially like that of Canada or Australia, with such autonomy and home rule
as are enjoyed by these free, self-governing colonies. Is not this demand just? Not only
the people of India, but many of the best Englishmen, answer unequivocally, Yes! In
the arduous struggle upon which India has entered to attain this end (arduous indeed
her struggle must be, for holders of autocratic and irresponsible power seldom in this
world surrender their power without being compelled) surely she should have the
sympathy of the enlightened and liberty-loving men and women of all nations.

-Atlantic Monthly magazine article (1909)

Document 7
We are meeting today in our session after fifteen months. The last session of the All-India
Muslim League took place at Patna in December 1938. Since then many developments have
taken place. ...But a great deal yet remains to be done. I am sure from what I can see and hear
that the Muslim India is now conscious, is now awake, and the Muslim League has by now
grown into such a strong institution that it cannot be destroyed by anybody, whoever he may
happen to be. Men may come and men may go, but the League will live for ever.

[[10] He [Gandhi] is fighting the British. But may I point out to Mr. Gandhi and the Congress
that you are fighting for a Constituent Assembly which the Muslims say they cannot accept;
which, the Muslims say, means three to one; about which the Mussalmans say that they will
never be able, in that way by the counting of head, to come to any agreementwnt which will be
real agreement from the hearts, which will enable us to work as friends; and therefore this idea of
a Constituent Assembly is objectionable, apart from other objections. But he is fighting for the
Constituent Assembly, not fighting the Mussalmans at all! ...

[[11]] So he wants the Constituent Assembly for the purpose of ascertaining the views of the
Mussalmans; and if they do not agree then he will give up all hopes, but even then he will agree
with us. (Laughter.) Well, I ask you. ladies and gentlemen, is this the way to show any real
genuine desire, if there existed any, to come to a settlement with the Mussalmans? (Voices of no,
no.) Why does not Mr. Gandhi agree, and.I have suggested to him more than once and I repeat it
again from this platform, why does not Mr. Gandhi honestly now acknowledge that the Congress
is a Hindu Congress, that he does not represent anybody except the solid body of Hindu people?
Why should not Mr. Gandhi be proud to say. "I am a Hindu. Congress has solid Hindu backing"?
I am not ashamed of saying that I am a Mussalman. (Hear, hear and applause.) I am right and I
hope and I think even a blind man must have been convinced by now that the Muslim League
has the solid backing of the Mussalmans of India (Hear, hear.) Why then all this camouflage?
Why all these machinations? Why all these methods to coerce the British to overthrow the
Mussalmans? Why this declaration of non-cooperation? Why this threat of civil disobedience?
And why fight for a Constituent Assembly for the sake of ascertaining whether the Mussalmans
agree or they do not agree? (Hear, hear.) Why not come as a Hindu leader proudly representing
your people, and let me meet you proudly representing the Mussalmans? (Hear, hear and
applause.)...

As regards other matters, we are still negotiating and the most important points are: (1) that no
declaration should be made by His Majesty's Government with regard to the future constitution
of India without our approval and consent (Hear, hear, and applause) and that no settlement of
any question should be made with any party behind our back (Hear, hear) unless our approval
and consent is given to it.

-Muhammad Ali Jinnah, Indian Muslim leader: Address to the Muslim League (1940)

END OF DOCUMENTS FOR QUESTION 1

WORLD HISTORY
SECTION II

Total Time- 1 hour, 30 minutes

Question 2 or Question 3

Suggested writing time: 35 minutes

Directions: Choose EITHER question 2 or question 3.


In your response you should do the following.

 Thesis: Present a thesis that makes a historically defensible claim and responds to all parts of
the question. The thesis must consist of one or more sentences located in one place, either in
the introduction or conclusion.
 Application of Historical Thinking Skills: Develop and support an argument that applies
historical thinking skills as directed by the question.
 Supporting the Arguments with Evidence: Utilizes specific examples of evidence to fully and
effectively substantiate the stated thesis or relevant argument.
 Synthesis: Extend the argument by explaining the connections between the argument and ONE
of the following.
 A development in a different historical period, situation, era, or geographical area.
 A course theme and/or approach to history that is not the focus of the essay (such as political,
economic, social, cultural, or intellectual history).
 A different discipline or field of inquiry (such as economics, government and politics, art history,
or anthropology).

3) Evaluate the extent to which the Enlightenment (1600's-1700's) marked a turning point in the triumph
of democratic revolutions in world history, analyzing what changed and what stayed the same from the
period before the Enlightenment to the period after it. (Historical Thinking Skill: Periodization)

or

2.) Evaluate the extent to which the Industrial Revolution marked a turning point in the history of
Europe's rise to material and commercial wealth.

In the development of your argument, explain what changed and what stayed the same from the period
immediately before the industrial revolution to the period immediately following it.. (Historical Thinking
Skill: Periodization)

STOP---END OF EXAM
AP
__________________________
Sample AP World History
2017 Multiple-Choice Answers
__________________________
Answers to Multiple-Choice Questions
1. (C) Trans-Atlantic Exchanges
Equiano describes being sold as a slave across the Atlantic Ocean.

2. (C) The abolition of international slavery


Equiano's writings influenced abolitionist movements in Europe.

3. (B) The Triangle Trade


The Triangle Trade is another term for the Trans-Atlantic slave trade.

4. (D) The urbanization of Europe during the Industrial Revolution


In Europe, industrialization, urbanization, and population growth were linked.

5. (A) Asia
The most populous nations are India, and China- two Asian countries.

6. (A) An increase in tensions between social classes

Tensions between capitalists and factory workers led to the rise of Communism.

7. (B) The beginning of an industrialized economy

The world's population has increased exponentially since Europe industrialized.

8. (C) women shouldn't enjoy full and equal rights with men

She is arguing for equal rights for women, contrary to popular opinion.

9. (A) The 19th Amendment to the U.S. Constitution, granting women the right to vote

The women's suffrage movement was influenced by the works of Olympe de Gouges.

10. (B) Radical supporters of the French Revolution

Many women resented the lack of change in status even after the revolution.

11. (D) The Mongols ability to conquer previously wealthy and powerful empires

The writings were in response to the rise of the Mongols after Gengis Khan.

12. (A) The military and political traditions of central Asian nomads

The political traditions of central Asian nomads allowed for brutality and slaughter.
13. (D) The spread of disease and eventual collapse of the Silk Road

The Mongols facilitated the spread of disease, leading to the deaths of millions.

14. (C) the rise of Zionism promoting Israel as a homeland for the Jews

The Balfour Declaration boosted calls of Zionism, and immigration to Israel.

15. (C) national boundaries

Zionism was an example of post war nationalism. The British supported Zionism.

16. (A) U.N. Israel/Palestine Partition Plan of 1947

The partition in 1947 was a continuity because it also divided Israel by Arabs and Jews.

17. (A) Sykes-Picot Agreement, granting Britain control of Mandate Palestine

Sykes-Picot gave control to Britain whereas the Balfour document favored Jews in Palestine.

18. (B) highlighting the decline of the natives as part of colonization

Most Spanish weren't concerned with the treatment of the native Americans.

19. (C) encourage the improved treatment of the natives

He's writing about the horrible loss of life in order to improve the situation.

20. (A) the treatment of slaves as part of the Trans-Atlantic Slave trade

The brutal demise of the Indians was similar to that of the slaves who came to the Americas.

21. (B) breaking free from the chains of colonization to establish an independent nation

Both passages deal with both authors' struggles for independence in a post-colonial world.

22. (A) show the state along with the people will influence political affairs

Sun Yet Sen was saying that the state would play an important role in society.

23. (D) the demand of colonial nations for independence and decolonization

Kenyatta's demands best reflect his early wants and desires for independence.

24. (B) the passage of democratic reforms under Communism and Dictatorship

The man was resisting communism in China, but influenced Soviet as well.
25. (D) the growing gap between Communist government and its people

In the late 1980's, the Soviet Union was crumbling and becoming more democratic.

26. (C) the government should be responsive to the needs of the people

The picture represented the people protesting against their governments.

27. (D) the government should ensure basic sanitation standards for workers

Before, nobody cared about hygiene and sanitation. Now, it's a public matter.

28. (A) the increase in social tensions and abuse of industrial workers

His writings showed sympathy for workers, and led to some of the first labor laws ever.

29. (D) Communists

Since he was concerned with the treatment of workers, communists should support him.

30. (C) debate over the treatment of Christian missionaries in East Asia

At first, Christianity was allowed in China. Then, Kangxi prohibited it for various reasons.

31. (D) The Boxer Rebellion

The Kangxi decree influenced the Boxer Rebellion in that both were anti-Christian.

32. (C) Christian missionaries

The Christian missionaries would not be able to support Kangxi's decree without a fight.

33. (B) The development of Christianity as a minority in China

The fact that China didn't officially promote Christianity led to its minority status.

34. (A) diversifying diets and thereby triggering a population increase

With the transfer of wheat and barley, the diets in the Americas changed.

35. (C) The population decline of Native Americans

The end of the passage mentions diseases, foreshadowing the decline of the Indians.
36. (A) The rise of the Triangle Trade

The Triangle Trade or Trans-Atlantic slave trade was part of the Columbian Exchange.

37. (D) The rising influence of Muslim governments and merchants.

The Abassids and other merchants sold goods from Aleppo to Malacca.

38. (A) promoted the advancement and diffusion of maritime technology

The spread of maritime technology like the compass was facilitated by sea routes.

39. (A) significance of understanding how to navigate the Monsoon winds

Explains why Indians needed to know the monsoon winds more than Mongolians.

40. (B) liberal revolutionaries

Liberal revolutionaries looking to establish democratic governments read Locke.

41. (B) the development of the Trans-Atlantic slave network

Locke said that no one ought to be deprived of life, yet America was founded on slavery.

42. (D) the drafting of the American Declaration of Independence

Locke influenced Thomas Jefferson when he wrote the Declaration of Independence.

43. (D) the role that religion has played historically in Persian culture

The passage is useful to read about how Greeks viewed Persian religion.

44. (A) the Persians have no images of gods nor altars for religious worship

The Greeks had many altars and places of worship for the gods like the Parthenon.

45. (D) The treatment of the Other upon first contact in the New World

The issue revolves around the treatment of natives by colonizers, and vice versa.

46. (A) the establishment of the encomienda system

In the earliest days, Indian labor was used on the encomienda system- slaves came later.
47. (C) Aztecs had no immunities to disease while the Europeans had superior metallurgy and

technology

As Jared Diamond explains, Europeans had the advantages of guns, germs, and steel.

48. (A) were the first successful attempt of the Romans to codify law

The Romans had many social customs but before, but it hadn't yet been codified.

49. (C) social tensions between the Patrician and Plebian classes

The lower class, Plebains, wanted to end the corruption and have a say in government.

50. (A) Political documents in modern law codes that reflect the principles of the Twelve

Tables

The best evidence to compare to modern day law would be to have cited examples.

51. (B) critique how imperialism operated in China

The document is using artistic devices to criticize how Europeans just took over China.

52. (D) inability of China to resist the imperialists

The poster shows the Chinese man standing behind the table with his arms up- defeated.

53. (A) The Japanese invasion of Manchukuo in the 1930's

The Japanese imperialist is at the table, just like the attack on China that led to WWII.

54. (C) the U.S. invasions of Afghanistan and Iraq

The invasions of Afghanistan and Iraq are policy decisions reflected in the speech.

55. (B) Franklin Delano Roosevelt

Much like Bush, F.D.R. declared a war after being attacked at Pearl Harbor.
AP
__________________________
Sample AP World History
2017 Free-Response Answers
__________________________
SECTION I, PART B: SHORT-ANSWER QUESTIONS (EXPLAINED)

Question 1

This question deals with comparing and contrasting religions in ancient


Mesopotamia and Egypt.

a) One similarity is that they both built architectural monuments that were
reserved for religious purposes. In Sumer, the inhabitants built ziggurat as a
form of temple worship. In Egypt, the pyramids served religious purposes
and burial grounds for pharaohs.

b) The main difference between the groups was their beliefs in the number
of gods. In Mesopotamia, the Hebrews were the first group to practice
monotheism- the belief in one G*d. On the other hand, the Egyptians were
polytheistic, believing in many gods. The Egyptians had a belief in the
afterlife, and worshipped many gods like Osiris and Isis whereas the Jewish
people worshipped only G*d.

c) One factor that accounts for the main difference is that in the ancient era
King David united Israel. Jerusalem became the capital and most holy city
for all Jews. This led to the full establishment of Jewish kingdoms in Israel
before Nebuchadnezar destroyed the Temple.
SECTION I, PART B: SHORT-ANSWER QUESTIONS (EXPLAINED)

Question 2

This question revolves around the map of the spread of Buddhism


during the Classical era.

a) There are many reasons for the spread of Buddhism during the time
period. Buddhism began in India before spreading throughout Asia. In
Afghanistan, the Kushan Empire promoted Buddhism and facilitated trade
along the Silk Road. Until the 21st century, there were giant Buddhist
statues in Bamyan, Afghanistan that were left over from the Kushan era. In
China, Faxian and Xuanzang in particular helped spread Buddhism into China
from India. They both went to India to learn about Buddhism before
spreading their teachings back in China. In Thailand, the followers practiced
a more conservative form of Buddhism, known as Theravada. In Korea and
Japan, they were both influenced by missionaries. The Silla Kingdom in
Korea promoted Buddhism as its official religion. More generally, the Silk
Road played a huge role in the spread of Buddhism. Most of the arrows on
the map are consistent with the trade routes of the Silk Road.

b) The biggest social effect has been that Southeast Asia is home to the
world's largest populations of Buddhist societies. They also have the highest
percentages of Buddhists in their countries. For example, China, Korea,
Burma, Thailand, and Japan all have large numbers of Buddhists in their
countries. They are in many ways Buddhist societies. In India, Ashoka built
stupas and promoted Buddhist policies.

c) In India, Buddhism didn't really penetrate into the southern regions which
remained mostly Hindus. Chinese society became Buddhist after Xuanzang
traveled to India and brought back Buddhism into China. Even though he
broke the Chinese law of leaving the country, he was welcomed back by the
emperor and made a court official. The Kushan Empire in Afghanistan
promoted Buddhism along the Silk Road.
SECTION I, PART B: SHORT-ANSWER QUESTIONS (EXPLAINED)

Question 3

The graph is at the center of this question. You need to analyze the
increase in the output of European manuscripts (books) from 500-1500 C.E..

a) The biggest historical factor for the increase in the output of manuscripts
in Europe from 500-1500 C.E. was the invention of the printing press. The
printing press was created by Johann Gutenberg in 1450. Gutenberg turned
the process of copying a book into a manufacturing process. Before his
invention, all of the books had to be copied by hand. That explains one
reason for the low output of manuscripts before the 15th century as seen on
the graph. Since the books had to be copied by hand, the process was
painstakingly slow.

There could have been other reasons as well like the Crusades or the
Silk Road. During the Crusades, from the 11th to 14th century there is an
increase on manuscripts shown on the graph. Perhaps, this was caused by
the exchange of Muslims and Christians during their Holy War which was
also promoted by the merchants during the 4th crusade. The Silk Road also
increased the spread of literature and travel accounts. For example, Marco
Polo's experiences on China were recorded and widely published in Europe.

b) The biggest historical effect was that knowledge flourished across Europe,
which later spread to the rest of the world. The invention of the printing
press made books much cheaper and more widespread. The levels of
education and literacy in Europe have skyrocketed as a result of the printing
press. Now, knowledge was for the common person and not just for the
political and religious elite. The growth of literature spread across Europe
during the Renaissance and has continued until the present day.
Shakespeare was the most famous Renaissance author, but there were
several others like Chaucer, Dante, and de Cervantes. Much of their works
have become classic novels and required reading for many students today.

c) Johann Gutenberg is not mentioned on the graph; however, he is the


most significant person in the increase in European manuscripts in the 15th
century. He invented the printing press, which led to the huge increase in
manuscripts from 250,000-500,000 in one century.

SECTION I, PART B: SHORT-ANSWER QUESTIONS (EXPLAINED)

Question 4

Here, you need to compare two passages about the role of science,
religion, and society.

a) The biggest difference is that the first passage expresses a desire and
need for experimental science whereas the second passage shows a belief in
religion as the highest order of law. Sir Francis Bacon claims we need to
have science that is based on experiments and inductive reasoning. The
second passage, from Galileo's trial, shows how authorities weren't
concerned with the progress of science so much as enforcing traditional
Catholic beliefs about the universe. The first passage is from a scientific
perspective, but the second one is from a Catholic point of view.

b) Sir Francis Bacon was most likely influenced by the Scientific Revolution.
He advocated for the advancement of science over religion in order for
society to progress. During the Scientific Revolution, scholars began using
science to prove natural phenomenon.

The documents from Galileo's trial are clearly from a Catholic perspective.
The Catholic Reformation may have played a huge role in convicting Galileo
of heresy. The believed that G*d's word was supreme, and trumped
scientific discovery, which was often viewed as heretical.

c) Copernicus' discovery of the Heliocentric Theory is one example of a


scientific discovery that was supported by Bacon, but opposed by the
Catholic authorities at Galileo's trial. Copernicus used his mathematical
models to determine that the sun was at the center of the universe. The
Catholic church explicitly opposed that belief. Galileo was on trial specifically
because he "held the doctrine that the sun is the center of the world." That
is exactly what Galileo was on trial for, holding form to Copernicus' belief.
The work of Copernicus was consistent with Bacon's belief that society
should be based on inductive reasoning.
SECTION II, PART A: DOCUMENT BASED QUESTION (EXPLAINED)

Document 1

This is a photo of Mohandas Gandhi and Jawaharlal Nehru sitting


together. Both of them were independence leaders of India involved in the
Indian National Congress, the main agency fighting for Indian independence.
The photo shows how close they were, and how they were constantly aware
of one another's positions. They look as though they are discussing and
contemplating the fight for independence.

Document 2

In this letter, Gandhi writes to Nehru about how he feels he is going to


be arrested that evening. Gandhi seems very calm. Most likely, he was being
arrested for his boycott of British salt. The letter is dated one day before the
Salt March when Gandhi and his followers collected salt despite the British
monopoly. It shows how hard they had to struggle to achieve independence.

Document 3

In this Indian politician's address to the Congress, he argues the need


for independence from the British. He claims that their biggest weapon is the
power of boycott, a non-violent tactic. The boycott of Indian goods was a
strategy of the Indian National Congress because it was non-violent yet
effective in hurting the British economically. He is trying to incite the
audience to demand their freedom immediately. He doesn't want a long term
approach to freedom.

Document 4

This is a letter from Nehru to his jailer. It shows that Nehru was also
jailed. It is similar to the Gandhi letter where he was imprisoned, but Nehru
is writing to express his discontent with the mistreatment of his mother. He
claims that the government insulted his mother and his wife. The document
doesn't give any details of boycott activities, but it does provide is with
insight into the animosity between the British and Indians during the era of
Indian nationalism.

Document 5

The picture is a newspaper article from the Amritsar Massacre in


Indian in 1919. It shows that the British reportedly killed 100 Indians. The
Amritsar Massacre showed how the British colonization of India was a violent
process. The massacre happened during the struggle for independence. It
was one thing that led to greater support for Indian independence.

Document 6

In this magazine article, the author explains what the struggle for
Indian independence is all about. The author argues the Indians have a right
to freedom. His article is trying to inform and persuade Western audiences
to support the cause for Indian independence. He says that India should be
granted the same status as Canada, or Australia.

Document 7

In this document, Muhammad Ali Jinnah criticizes Gandhi and explains


the need for a Muslim country. The author is addressing his own
organization, the Muslim League, and telling them that Gandhi is only
fighting for Hindu independence. He claims the Muslims will not be
adequately represented in the new Hindu majority government if Gandhi has
his way. That's why he believes they need a separate country, Pakistan,
where all the Muslims can live together in peace. In the end, he remarks
that the Muslims won't accept any negotiation regarding India's
independence unless they approved it. His address stirred up anti-Hindu
sentiments and promote Muslim nationalism in Pakistan.
SECTION II, PART B: LONG ESSAY QUESTION (EXPLAINED)

Question 2

This question asks you to write an essay evaluating the extent to


which the Enlightenment marked a turning point in the history of
democratic revolutions. The first thing to know is that there were three
democratic revolutions in America, France, and Haiti. Now, you need to know
some facts and concepts about each one that you write about.

(IDEAS & FACTS TO MENTION IN YOUR ESSAY)

 Before the Enlightenment, monarchy was the rule of the day.

 Europe was ruled by mostly monarchies, especially Britain and France.

 France controlled the island of Haiti as a colony.

 France's social system consisted of three estates or classes.

 The third estate was highly taxed, but not the other estates.

 All revolutions were influenced by Locke's ideas of the social contract.

 Locke also argued for 3 inalienable rights- life, liberty and property.

 In Haiti, the revolutionaries had a successful slave rebellion.

 In America, the colonists overthrew the British monarchy.


 The Americans established the first democratic republic of the era.

 America established a government based on Enlightenment ideas,

 Montesquieu, Voltaire and Rousseau also influenced democracy.

 The Enlightenment was the intellectual roots of democratic revolutions

SECTION II, PART B: LONG ESSAY QUESTION (EXPLAINED)

Question 3

It is important to keep in mind that Europe had transitioned out of the


Dark Ages during the Renaissance in the 1400's. The industrial revolution
usually is referred to as roughly the time period from 1750-1900.

(IDEAS & FACTS TO MENTION IN YOUR ESSAY)

 Before the Renaissance, Europe was in the Dark Ages.

 The Renaissance was a rebirth of Greco-Roman culture in Europe.

 Europe was becoming rich again, and catching up to the East.

 After the IR, Europe became richer and more powerful than the East.

 The steam engine by James Watt was a significant factor in the IR.

 Europe became more urbanized, and less rural.

 The living standard of the average European improved greatly.

 The industrial revolution led to the era of imperialism.

 Industrialization and imperialism went hand in hand.

 After the era of industrialization, Europe was able to conquer China.


 The Opium Wars showed Britain's industrial strength.

 The IR led to the rise of a middle class in Europe as opposed to serfs.

 The rise of wealthy capitalists was a result of the IR as well.

AP
__________________________
Sample AP World History
2017 Multiple Choice Questions
__________________________
WORLD HISTORY

SECTION I, Part A

Time—55 minutes

55 Questions

Directions: Each of the questions or incomplete statements below is followed by four suggested answers or
completions. Select the one that is best in each case and then fill in the appropriate letter in the corresponding space
on the answer sheet.

Questions 1 - 3 refer to the excerpt below.

“Another of their (people of Mali's) good qualities is their habit of wearing clean white garments on Fridays. Even if
a man has nothing but an old worn shirt, he washes it and cleans it, and wears it to the Friday service. Yet another is
their zeal for learning the Koran by heart. They put their children in chains if they show any backwardness in
memorizing it, and they are not set free until they have it by heart. I visited the qadi in his house on the day of the
festival. His children were chained up, so I said to him, "Will you not let them loose?" He replied, "I shall not do so
until they learn the Koran by heart."
Ibn Battuta, Travels in Africa, c. 1325

1) Ibn Battuta's account suggests that


cultural encounters between Africans and
Arabs in the 1300's was most directly
shaped by

(A) interest in religious debate

(B) Trans-Atlantic Exchanges

(C) cross-cultural trade

(D) traditions and practices of Islam


2) Ibn Battuta's account contributed most
directly to which of the following trends?

(A) The rise in social tensions between


African rulers and their people
(B) greater political autonomy for Africa (A) The development of the Silk Road

(C) the rise and spread of Islam (B) The urbanization of African cities

(D) the rise of child soldiers in Africa (C) The development of African literature

(D) The tension within Muslim sects

3) The experience described in the excerpt


is an example of which of the
following historical developments?

Questions 4 - 7 refer to the graph below.

GROWTH OF THE URBAN POPULATION IN CHINA, 2001 C.E..- 2009 C.E

4. Which of the following was a (D) The large scale migration of Chinese
significant cause of the trend from 2001 into America and Europe
to 2009 shown in the graph?

(A) Active encouragement of export


and manufacturing for the world
economy

(B) the rising rates of poverty in the


5. The people represented by the
urban areas
graph from 2001-20095 most typically
(C) Incentives offered by European worked in what type of employment?
companies looking to hire foreign
(A) high finance and real estate
migrants from colonial territories
(B) government controlled industries (C) the promotion of agricultural reform
to increase land ownership for many
(C) manufacture and export
(D) The removal of government from
(D) support services for the tourism all aspects of the economy

7. The main trend shown in the graph


was most directly associated with which
of the following processes occurring in
the global economy at the time?

(A) The convergence of world


markets in the age of globalization
6. Which of the following was a direct
effect of the trend in population growth (B) The abolition of tariffs for nations
between 2001-2009 shown on the that support capitalist policies
graph?
(C) The increase in the number of
(A) urban poverty rates have transnational corporations from Asia
skyrocketed
(D) The increase in exports of
(B) the environment in urban areas has manufactured goods from America
become polluted and hazardous
Questions 8 - 10 refer to the excerpt below

"To account for and excuse the tyranny of man, many ingenious arguments have been brought forward to
prove that the two sexes, in the acquirement of virtue, ought to aim at attaining a very different character: or to speak
explicitly, women are not allowed to have sufficient strength of mind to acquire what really deserves the name of
virtue... Women are told from their infancy, and taught by the example of their mothers, that a little knowledge of
human weakness, justly termed cunning, softness of temper, outward obedience, and a scrupulous attention to a
puerile kind of propriety, will obtain for them the protection of man; and should they be beautiful, every thing else is
needless, for at least twenty years of their lives..."

Mary Wollstonecraft, Vindication of the Rights of Woman, 1792

8. The sentiments conveyed in the


excerpt most directly opposed the
prevailing ideal in the early
eighteenth century that

(A) since ancient times women have


9. Which of the following developments
been oppressed
in the first half of the twentieth century
(B) women should be more like their best represented the continuation of
mothers the ideas expressed in the
excerpt?
(C) patriarchy was viewed as legitimate
(A) The 19th Amendment to the U.S.
(D) women should educate their Constitution, granting women the right
children about the rights and to vote
responsibilities of citizenship
(B) Women’s support for WWI

(C) Support for outlawing the


production and sale of alcohol

(D) A movement focused on religious


revivals and personal conversion
10. Many female supporters of the (C) the failure by many liberals to
Enlightenment opposed many grant women full equality and
philosophes over what issue? citizenship

(A) the abolition of slavery (D) the decline of women's traditional


roles in society
(B) the treatment of Native Americans

Questions 11 - 13 refer to the excerpts below.

"... The torture of a criminal during the course of his trial is a cruelty consecrated by custom in most nations...No
man can be judged a criminal until he be found guilty; nor can society take from him the public protection until it
have been proved that he has violated the conditions on which it was granted. What right, then, but that of power,
can authorize the punishment of a citizen so long as there remains any doubt of his guilt?...Either he is guilty, or not
guilty. If guilty, he should only suffer the punishment ordained by the laws, and torture becomes useless, as his
confession is unnecessary. If he be not guilty, you torture the innocent; for, in the eye of the law, every man is
innocent whose crime has not been proved..."
Cesare Beccarria, On Crimes and Punishment, 1764

“Excessive bail shall not be required, nor excessive fines imposed, nor cruel and unusual punishments inflicted.
8th Amendment, U.S. Constitution, 1791

(B) The belief in Absolutism

11. The excerpts were written in (C) The principle of popular sovereignty
response to the
(D) The ideas of the Enlightenment
(A) the American Revolution

(B) the debates between conservatives


and radicals over lengthy prison
sentences
13. The ideals expressed in both
(C) the changing attitudes among documents has accounted for which of
European monarchies to reform prisons the following features of the
democratic criminal justice systems
(D) political and social acceptance of since the Enlightenment era?
torture as a means of discipline and
punishment for suspected criminals (A) The increase in the rights of the
accused like innocent until proven
12. The ideas about torture and guilty
punishment expressed by both
passages are most consistent with (B) the decrease in beheadings during
which of the following? the French Revolution

(A) The concept of Constitutionalism (C) the conflict of the checks and
balances in a democracy
(D) The growth in conflict between the
wealthy colonists and the slaves

Questions 14 - 17 refer to the excerpts below.

“1.) ...France and Great Britain are prepared to recognize and protect an independent Arab State
or a Confederation of Arab States in the areas (A) and (B) marked on the annexed map, under the
suzerainty of an Arab chief. That in area (A) France, and in area (B) Great Britain, shall have
priority of right of enterprise and local loans. That in area (A) France, and in area (B) Great
Britain, shall alone supply advisers or foreign functionaries at the request of the Arab State or
Confederation of Arab States...
Paul Cambon's letter to Edward Grey, Sykes-Picot Agreement, 1916

14. Which of the following was the


most immediate result of the
declaration?

(A) Arab nationalist became


increasingly united over in their political
strategies

(B) the rise in concern about the


persecution of Jews in Europe 15. The decision excerpted most
directly reflected a growing belief after
(C) the Arab territories of Transjordan,
the First World War that the Arab
Palestine, Syria, and Lebanon were
world's political borders should be
divided among Europeans
divided according to
(D) The British refused to support the
(A) racial boundaries
idea and granted full independence to
all Arab nations (B) religious preference

(C) national boundaries

(D) a mandate system under European


influence
(B) Berlin Conference

(C) Yalta Conference

(D) Treaty of Versailles

17. The Sykes-Picot Agreement stood in


contrast mostly to which of the
following other decisions?

(A) Balfour Declaration

(B) 14 Points Plan

(C) Berlin Conference


16. Which of the following events
demonstrated the strongest similarity (D) The establishment of the League of
with the idea expressed in the excerpt? Nations
(A) 14 Points Plan
Questions 18 - 20 refer to the excerpt below.

" ...The trade of slaves is in a more peculiar manner the business of kings, rich men, and prime merchants, exclusive
of the inferior sort of Blacks....This barbarous usage of those unfortunate wretches, makes it appear, that the fate of
such as are bought and transported from the coast to America, or other parts of the world, by Europeans, is less
deplorable, than that of those who end their days in their native country; for aboard ships all possible care is taken to
preserve and subsist them for the interest of the owners, and when sold in America, the same motive ought to prevail
with their masters to use them well, that they may live the longer, and do them more service. Not to mention the
inestimable advantage they may reap, of becoming Christians, and saving their souls, if they make a true use of their
condition....

John Barbot, How Slaves Were Acquired , 1732

18. The ideas expressed in the excerpt


differed from the predominant
abolitionist views of the slave trade
because the passage

(A) lauds the possible benefits of the


19. The excerpt best reflects an effort
slave trade
by Barbot to
(B) highlights the decline in health
(A) convert the slaves to Christianity
for the slaves
(B) persuade other abolitionists to join
(C) promotes new laws that would
him
make the slave trade illegal
(C) encourage the improved treatment
(D) encourages the slave owners to be
of the slaves
good Christians
(D) prevent the abuses of slavery
from spreading to the Americas

20. Which of the following best


represents a change in the years after
1800 with the account that Barbot
described in the excerpt?

(A) the Berlin Conference


(B) the success of the Haitian (D) Simon Bolivar's struggle to unify
Revolution South America into a gran Colombia

(C) The European colonization of Africa


during the era of high imperialism

Questions 21 - 23 refer to the excerpts below.

"Damascus (city in Syria) is the paradise of the east and the rising place of its radiant beauty. The city was highly
honored when Allah gave refuge to the Messiah and his Mother (Muslims believe Jesus and Mary sought refuge in
Damascus) on an 'elevated spot, secure and abundant in water' {Sura 23:52). In fact wherever you look in your gaze,
by Allah, they told the truth who said, "If Paradise be on earth, Damascus must be it; if it is m Heaven, Damascus
can parallel and match it."
Source: Ibn Jubayr, Spanish Muslim traveler and geographer, 1184 CE

"Cordoba after the Arab conquest of the Iberian Peninsula became the ultimate goal of the land, headquarters of the
army, mother of all towns, seat of the virtuous and pious, and abode of the people of intellect and learning...The
explanation of the superiority of the Cordobans over all others past and present lies in the fact that the city's climate
is one of research and investigation in the variety of sciences and literature."
Source: Al-Hijari, Andalusian (Spanish Muslim of Arab origin) historian, from his writings, 1188

21. The statements of both Muslim


authors share the same goal of?

(A) encouraging non-Muslims to come


to Arabia and convert to Islam

(B) encouraging loyal Muslims to join 22. Ibn Jubayr issued the writings
the Crusades primarily in order to

(C) spreading the ideals of Islam across (A) show from a Muslim perspective
the entire region the splendors of an Islamic city

(D) portraying the cities of Islam as the (B) gain support from Muslims in far off
most virtuous places on Earth lands like India and China

(C) protect Muslim interests in the


Middle East

(D) support the expansionist policies


of the Umayyad Dynasty

23. Al-Hijari's writing best reflects


which of the following developments
in the spread of Islam?
(A) the expansionist policies of the (C) the contrast between Islamic and
Umayyad Christian cities

(B) the philosophical and scholarly (D) the demand of Muslims to join the
atmosphere of Islamic Spain Crusades to defeat the Christians

Questions 24 - 26 refer to the late 20th century photograph by Stuart Franklin.

24. Conditions like those shown in the (D) The decline of Communist
image contributed most directly to operations in East Asia
which of the following?

(A) an increase in the spread of U.S.


military bases in places like Okinawa

(B) an increase in the rise of 25. The conditions shown in the image
Communist party operations depict which of the following trends in
the mid 20th century?
(C) a decline in the use of American
troops to spread democracy (A) the growing division between rich
and poor in society
(B) the rise of a counter-culture
movement

(C) the use of military force as a means


of promoting democracy

(D) the rise of the United Nations

26. Advocates for individuals such as


those shown in the image would have
most likely agreed with which of the
following perspectives?

(A) the establishment of the United


Nations by the Atlantic Charter

(B) Capitalism, free of all government


regulation, would improve society

(C) collective security would lead to too


many conflicts

(D) Democracy was at risk, and should


be protected at all costs
Questions 27 - 29 refer to the excerpt below.

" Papal indulgences for the building of St. Peter's are circulating under your most distinguished name, and
as regards them, I do not bring accusation against the outcries of the preachers, which I have not heard, so much
as I grieve over the wholly false impressions which the people have conceived from them; to wit, -- the unhappy
souls believe that if they have purchased letters of indulgence they are sure of their salvation; again, that so soon
as they cast their contributions into the money-box, souls fly out of purgatory; furthermore, that these graces [i.e.,
the graces conferred in the indulgences] are so great that there is no sin too great to be absolved, even, as they say
-- though the thing is impossible -- if one had violated the Mother of God; again, that a man is free, through these
indulgences, from all penalty and guilt."

Martin Luther, Letter to Archbishop of Mainz, 1517

27. Which of the following aspects of


Luther's description expresses a
major change in Europeans' views of
Catholicism?

(A) The idea that indulgences can be


28. Luther's ideas are most directly a
sold but only by the Pope
reaction to the
(B) The idea that religion has a role to
(A) the increasing sentiments that the
play in ensuring economic codes
Catholic church was interested
(C) The idea that salvation can be in money more than the gospel
reached by following the sacraments
(B) growing backlash of Catholics
(D) The idea that the Pope and against the Protestants
Catholic church were corrupt and in
(C) the increase in demand for new
need of a religious reformation
Christian converts

(D) the persecution of religious


minorities during the Inquisition

29. Luther's position regarding the


papal sale of indulgences was most
strongly supported by which of the (B) members of the Spanish Crown
following?
(C) the Catholic Church
(A) Italian merchants
(D) Calvinists

Questions 30-33 refer to the excerpt below.

"If a woman bring about the death of her husband for the sake of another man, they shall impale her.

If a son strike his father, they shall cut off his fingers.
 
If a man destroy the eye of another man, they shall destroy his eye.
 
If one break a man’s bone, they shall break his bone...
 
If a man knock out a tooth of a man of his own rank, they shall knock out his tooth...
 
If he be a freeman, he (the physician) shall receive five shekels...
 
If a physician operate on a man for a severe wound with a bronze lancet [surgical knife] and cause the man’s death;
or open an abscess (in the eye) of a man with a bronze lancet and destroy the man’s eye, they shall cut off his
fingers..."
Hammurabi, Law Code (translated), 1,800 B.C.E. 

30. The issues expressed by Hammurabi (C) the refusal to protect minority
were a response to the? groups such as women and slaves

(A) controversy regarding legal support (D) The development of free public
for various minority religions health care

(B) the need to develop a uniform and


public law code to unify his people
32. The excerpt would be most useful to
(C) debate over the treatment of historians as a source of information
women about which of the following

(D) the dispute over monotheism (A) The role that law codes play in
ancient Mesopotamian people's lives

(B) the relative equality of all citizens


31. The ideas expressed in Hammurabi's under the protection of the law in
law code most strongly influenced ancient Mesopotamia
which trend in ancient Mesopotmia?
(C) the harsh realities of life for women
(A) The establishment of legal codes under a patriarchal society
(B) the equal treatment of women (D) The interaction between
throughout Mesopotamia Mesopotamian rulers and his people
33. Which of the following (B) The idea of equal pay for women
developments demonstrated the
strongest continuity with the idea (C) The need for no cruel and unusual
expressed in the decree? punishment

(A) Modern democracies are based on (D) The separation of religious and
the rule of law political law codes

Questions 34 - 36 refer to the diagram below.

Courtesy of Wikicommons

34. The import of Old World crops to


the New World transformed American
society mostly by?

(A) diversifying diets and thereby


triggering a population increase

(B) encouraging the privatization of


35. The patterns depicted in the
lands
diagram most directly foreshadowed
(C) improving relations between which of the following developments?
Europeans and native inhabitants
(A) the spread of corn from South
(D) encouraging warfare between America to North America
European powers
(B) the gradual transition from a feudal
to capitalist society
(C) The population decline of Native
Americans

(D) the emergence of racially mixed


populations in the Americas

36. The trends portrayed in the diagram


most closely corresponds with which of
the following major historical
developments in the Atlantic world?

(A) The rise of the Triangle Trade

(B) The Manila Galleons

(C) The growth of imperialism in Africa

(D) The Plague


Questions 37 - 39 refer to the map below.
The Spread of Islam 622-750 C.E.

37. The pattern of Islamic conversions (D) The Byzantine government's


after 622 C.E. were linked most directly inability to impose strict control in its
by which of the following factors? territories

(A) the large size of the Mediterranean


Sea in relation to Arabia
38. The difference in conversion
(B) the expansionist policies of the patterns from the first four caliphs to
first four caliphs and the Umayyad that of the Umayyads had which of the
Dynasty following effects?

(C) the production of luxury goods to be (A) the promotion of science,


sold in Asia along the Silk Road literature, and philosophy in Spain
(B) a decrease in the settlements along
the Mediterranean

(C) the decline in the importance of


Mecca as a holy city

(D) a major increase in trade with China

39. The difference in trade patterns


between the land and sea routes best
explains the

(A) significance of understanding how


to navigate the Monsoon winds

(B) development of religious


differences between Africans, and
Persians

(C) spread of the Bubonic Plague

(D) growth of religious tensions


between Christians and Muslims
Questions 40 - 42 refer to the excerpt below.

"... Upon this a question arises: whether it is better to be loved than feared or feared than loved? It may be answered
that one should wish to be both, but, because it is difficult to unite them in one person, it is much safer to be feared
than loved,... men have less scruple in offending one who is beloved than one who is feared, for love is preserved by
the link of obligation which, owing to the baseness of men, is broken at every opportunity for their advantage; but
fear preserved you by a dread of punishment which never fails... Nevertheless a prince ought to inspire fear in such a
way that, if he does not win love, he avoids hatred; because he can endure very well being feared whilst he is not
hated, which will always be as long as he abstains from the property of his citizens and subjects and from their
women.

Niccolo Machiavelli, The Prince, 1513

40. Which of the following groups


would have been most likely to support
Machiavelli's views expressed in the
excerpt?

(A) Conservative Democrats


42. In the 20th century, the views
(B) an elected Senator expressed by Machiavelli most directly
contributed to
(C) Absolute monarchs
(A) increased political tension during
(D) the Pope the Cold War

(B) the use of violence to achieve


colonial independence
41. Which of the following most directly
undermines Machiavelli's claims? (C) the rise of fascism and dictatorship
throughout Europe
(A) the appeal of free and democratic
elections in various countries (D) the rising tide of nationalism that
swept across the Middle East after WWI
(B) the public support for Nazism gained
by Adolf Hitler

(C) the rule of Joseph Stalin in the


Soviet Union

(D) a majority of Enlightenment


philosophes were deists
Questions 43 - 44 refer to the excerpt below.

"The Persians do not eat the flesh of cows and calves, but mutton to a vast extent and horseflesh, which is
the most esteemed and by the nobles... Their garb is a long garment, different from that of the Turks: they tie shawls
round their waists, and almost all of them go clothed in cotton stuffs of various colours in imitation of the king.
Their chief food is rice with meat, and they do not use such variety, nor dainties as in these countries [of Europe]:
and they are frugal and satisfied with little food... Almost all of them drink wine: they sit and eat on the ground... on
rich carpets... On the street side they have no windows, so that their women should not be seen: and thus the streets
are not attractive, nor is the city fine... There are some of them, who profess to be philosophers and mathematicians,
almost all of them to be poets: and they continually have books in their hands. They have many large mosques,
where they go to say their prayers, and they allow any nation whatsoever to enter them..."
Father Simon, Report on Persia, c. 1604 C.E.

43. The excerpt would be most useful to


historians as a source of information
about which of the following?

(A) the harsh realities of living under 44. Which of the following was a major
such a repressive regime contrast between the customs of the
Persians and those of the Turks?
(B) the role that food plays in the
everyday lives of Safavid Muslims (A) the Ottomans had cross-cultural
interactions with Europeans
(C) the conflicts between Europeans
and Persians in the early modern era (B) the Ottomans were considered one
of the main Islamic gunpowder empires
(D) the cultural customs of the Persians
under the Safavid Dynasty (C) the Persians practiced Shi'a Islam,
which venerated the lineage of Ali

(D) the Persians promoted law and


education
Questions 45 - 47 refer to the excerpt below.

" Govern the state by correctness;


Deploy the army by deception;
Acquire the empire by taking no action (wushi).
How do I know this is so?
By this.
The more prohibitions there are in the world, The poorer are the people.
The more sharp weapons the people have, The more disorder is fomented in the family and state.
The more adroit and clever men are, The more deceptive things are brought forth.
The more laws and ordinances are promulgated, The more thieves and robbers there are.
."
Laozi, Daodejing, c. 500 B.C.E

45. The Daoist account most directly


illustrated the debates about which of
the following issues in classical
China?

(A) The limits of the government


46. The attitudes of prohibitions
(B) The proper role of government described by Laozi is most consistent
with which of the following
(C) The social tensions between farmers developments in the 20th century?
and landlords
(A) The Great Leap Forward and
(D) The clash between military and Cultural Revolution
government officials
(B) the Soviet Union during the Stalin
era

(C) the independence of India

(D) the transition of a communist


country to capitalism
47. Efforts by supporters of Laozi to
gain official promotion by the Han
government ultimately failed because

(A) the Han rulers favored laws more


based on harmony with nature

(B) Daoists weren't interested in


gaining s converts

(C) the popularity of other philosophies


like Confucianism, and Buddhism
among popular and elite forces

(D) The Daoists were always plagued


with internal power struggles
Questions 48 - 50 refer to the excerpt below.

“When the war in Kalinga was over, and all the people were conquered, he [Ashoka] felt inside him a great crisis, a
stirring for meaning and a remorse. Ashoka goes on a pilgrimage seeking a guru, a teacher. And by the riverbank he
met a Buddhist monk and the monk told him to sit beneath the Bodhi tree where the Buddha had found
Enlightenment. And there the power of ideas and the power of the state came together in a uniquely Indian way. [It
was] a rejection of the path of violence [and] of a whole way of understanding history. While he was here, Ashoka
gave rich gifts to the poor. He consulted with local communities about proper governance, about good
conduct...forming in his mind now as a political order the sort of which had never been conceived of before in the
history of the world."

- Michael Wood, PBS's Story of India (2009)

48. The Battle of Kalinga was significant


in the history of India because it

(A) was the first major battle against a


neighboring state 49. The pilgrimage taken by Ashoka
most directly reflected?
(B) led to the decline of trade
(A) a desire to expand territory
(C) the most violent conflict up to that
point (B) the desire to put philosophical and
political theory into practice
(D) it was a turning point in the
promotion of Buddhism (C) social tensions between the Hindus
and Buddhists

(D) to spread Buddhism across Asia

50. Which of the following evidence


would best support Wood's argument
in the excerpt?

(A) Military documents from all of the


major civilizations up to that point

(B) Testimony from government


officials that Ashoka was
influential on modern politics
(C) Narratives from the lives of Indians (D) Statistical data revealing the
who and designed the Indian national number of Buddhists in India before
flag and after Ashoka's pilgrimage

Questions 51 - 53 refer to the political cartoon below.

Courtesy of Columbia University

51. The poster was intended to

(A) persuade man to read more diverse


types of literature

(B) promote the ideals of the Chinese


communist revolution

(C) advocate for the promotion of men 52. The poster most directly reflects
and women in government positions the

(D) convince peasants to join the (A) emergence of China as an industrial


Cultural Revolution power
(B) mobilization of all social classes in
support of the Cultural Revolution and
it's omnipotent leader

(C) the victory over the defeat of Japan


after WWII

(D) the diversity of opinions during the


Cultural Revolution

53. Which of the following represents a


later example of a change in policy
from the campaign highlighted in the
poster?

(A) The restrictions on the internet

(B) The government's decision to limit


religious freedom

(C) China's liberalization of the


economy after Deng Xiaoping opened
the economy to western business

(D) China's transition to a


manufacturing economy
Questions 54 - 55 refer to the excerpt below.

" It is true that South Africa was often brought to the brink of destruction because of
differences... Since we have achieved our freedom, there can only be one division amongst us: between
those who cherish democracy and those who do not! As freedom loving people, we want to see our
country prosper and provide basic services to all. For our freedom can never be complete or our
democracy stable unless the basic needs of our people are met...As we rebuild our country, we should
remain vigilant against the enemies of development and democracy, even if they come from within our
own ranks. Violence will not bring us closer to our objectives. All of us should ask ourselves the question:
Have I done everything in my power to bring about lasting peace and prosperity in my city and my
country? ...

Nelson Mandela, On Reconciliation, 04/16/1999

54. The excerpt best reflects an effort 55. The ideas expressed by Mandela in
by Mandela to encourage? the excerpt were most similar to those
of which twentieth century world
(A) the landowning whites to give up leader?
some of their land
(A) Mikhail Gorbachev
(B) promote unity and integration in a
post-Apartheid South Africa (B) Ho Chi Minh

(C) the enactment of socialist programs (C) John F. Kennedy


for all social services so as to make
them dependent upon the government (D) Mahandas Gandhi

(D) the construction of a democratic


political system based on the two party
system
END OF PART A
IF YOU FINISH BEFORE TIME IS CALLED, YOU MAY
CHECK YOUR WORK ON PART A. DO NOT GO ON TO
PART B UNTIL YOU ARE TOLD TO DO SO.
AP
__________________________
Sample AP World History
2017 Free-Response Questions
__________________________
Directions: Read each passage carefully. Answer on a separate paper using
complete sentences- bullet points are not acceptable. 4 questions; 50 minutes.

1) Answer a, b, and c
a) Briefly explain ONE important similarity between the Bantu and Meso-
American migrations in populating their respective continents prior to 500 B.C.

b) Briefly explain ONE important difference between the Bantu and Meso-
American migrations in populating their respective continents prior to 500 B.C.

c) Briefly explain ONE way in which the difference you indicated in b contributed
to a difference in the development of pre-Columbian civilizations in the
Americas or traditional African civilizations.
Use the diagram below to answer all parts of the question that follows.

"Woman is a violent and uncontrolled animal... If you allow them to achieve complete equality with men,
do you think they will be easier to live with? Not at all. Once they have achieved equality, they will be your
masters."
Source: Cato the Elder, Speech in the Roman Senate (195 B.C.)

"Why should we pay taxes when we do not share in the offices, honours, military commands, nor, in short,
the government, for which you men fight between yourselves, with such harmful results?"

Source: Public Speech given by group of Roman women

2. a) Describe ONE important difference between the views of women and


citizenship expressed in the two passages.

b) For EACH of the passages, identify and explain ONE factor (such as a historical
development, an intellectual or philosophical trend, or a religious belief) that
likely informed the view of women and citizenship expressed in the passage.
Answer all parts of the question that follows.

3.Many historians argue that the Mongol conquest of Baghdad in 1258 was a
significant turning point in world history.

a) Identify ONE specific piece of evidence that supports the argument, and
explain how the piece supports the contention.

b) Identify ONE specific piece of evidence that undermines the argument, and
explain how it undermines the contention.

c) Briefly explain how ONE person, event, or development from the period 500-
1500 could be used to support the argument.
Use the passage below to answer all parts of the question that follows.

"... No one ought to harm another in his life, health, liberty, or possessions... ; for
without this the law could not have that which is absolutely necessary to its being a law,
the consent of the society, over whom nobody can have a power to make laws but by
their own consent and by authority received from them….: They must not raise taxes on
the property of the people without the consent of the people given by themselves or their
deputies.... When any one, or more, shall take upon them to make laws whom the people
have not appointed so to do, they make laws without authority, which the people are not
therefore bound to obey; by which means they come again to be out of subjection, and
may constitute to themselves a new legislative, as they think best, being in full liberty to
resist the force of those who, without authority, would impose anything upon them…."

John Locke: Two Treatises of Government (1689)

1. a) Briefly explain ONE implication for public policy of Locke's view on society.

b) Identify and explain ONE factor (such as a historical development, an


intellectual or philosophical trend, or a religious belief) that likely informed the
view of authority and society expressed in the passage.

c) Briefly explain how ONE person, event, or development from the period
1750-1914 that is not explicitly mentioned in the excerpts could be used to
support Locke's interpretation.

END OF SECTION 1
END OF SECTION 1

IF YOU FINISH BEFORE TIME IS CALLED, YOU MAY


CHECK YOUR WORK ON PART A. DO NOT GO ON TO
PART B UNTIL YOU ARE TOLD TO DO SO.
WORLD HISTORY

SECTION II

Total Time-- 1 hour, 30 minutes

Question 1 (Document-Based Question)

Suggested reading and writing time: 55 minutes

It is suggested that you spend 15 minutes reading the documents and 40 minutes writing your
response.

Note: You may begin writing your response before the reading period is over.

Directions: Question 1 is based on the accompanying documents. The documents have been edited for
the purpose of this exercise.

In your response you should do the following:

Thesis: Present a thesis that makes a historically defensible claim and responds to all parts of the
question. The thesis must consist of one or more sentences located in one place, either in the introduction
or the conclusion.

Argument Development: Develop and support a cohesive argument that recognizes and accounts for
historical complexity illustrating relationships among historical evidence such as contradiction,
collaboration, and/or qualification.

Use of the Documents: Explain the significance of the author's point of view, author's purpose, historical
context, and/or audience for at least four documents.

Contextualization: Situate the argument by explaining the broader historical events, developments, or
processes immediately relevant to the question.

Outside Evidence: Provide an example or additional piece of specific evidence beyond those found in
the documents to support or qualify the document.

Synthesis: Extend the argument by explaining the connections between the argument and ONE of the
following.

 A development in a different historical period, situation, era, or geographical area.


 A course theme and/ or approach to history that is not the focus of the essay (such as political,
economic, social, or intellectual history)
 A different discipline or field of inquiry (such as economics, government and politics, art history,
or anthropology).

1. Explain the causes of the rise of the worker's rights movement in Europe during the period 1750-1914.
Document 1

Factory Wages in Lancashire, England in 1830


Age of Worker Male Wages Female Wages
under 11 2s 3d. 2s. 4d.
11 - 16 4s. 1d. 4s. 3d.
17 - 21 10s. 2d. 7s. 3d.
22 - 26 17s. 2d. 8s. 5d.
27 - 31 20s. 4d. 8s. 7d.
32 - 36 22s. 8d. 8s. 9d.
37 - 41 21s. 7d. 9s. 8d.
42 - 46 20s. 3d. 9s. 3d.
47 - 51 16s. 7d. 8s. 10d.
52 - 56 16s. 4d. 8s. 4d.
57 - 61 13s. 6d. 6s. 4d.
Source: Spartacus Educational.com

Document 2
Betty Harris, age 37: I was married at 23, and went into a colliery when I was
married. I used to weave when about 12 years old; can neither read nor write. I work
for Andrew Knowles, of Little Bolton (Lancs), and make sometimes 7s a week,
sometimes not so much. I am a drawer, and work from 6 in the morning to 6 at night.
Stop about an hour at noon to eat my dinner; have bread and butter for dinner; I get
no drink. I have two children, but they are too young to work. I worked at drawing
when I was in the family way. I know a woman who has gone home and washed
herself, taken to her bed, delivered of a child, and gone to work again under the week.

I have a belt round my waist, and a chain passing between my legs, and I go on my
hands and feet. The road is very steep, and we have to hold by a rope; and when there
is no rope, by anything we can catch hold of. There are six women and about six boys
and girls in the pit I work in; it is very hard work for a woman. The pit is very wet
where I work, and the water comes over our clog-tops always, and I have seen it up to
my thighs; it rains in at the roof terribly. My clothes are wet through almost all day
long. I never was ill in my life, but when I was lying in.

My cousin looks after my children in the day time. I am very tired when I get home at
night; I fall asleep sometimes before I get washed. I am not so strong as I was, and
cannot stand my work so well as I used to. I have drawn till I have bathe skin off me;
the belt and chain is worse when we are in the family way. My feller (husband) has
beaten me many a times for not being ready. I were not used to it at first, and he had
little patience...

- Testimony from a woman worker to the British Government (1842)


Document 3

Q: 'What was the effect of this state of the work-places upon the habits of the workmen?
A: It had a very depressing effect on the energies;...The natural effect of the depression
was, that we had recourse to drink as a stimulant. We went into the shop at six o'clock
in the morning; but at seven o'clock, when orders for the breakfast were called for, gin
was brought in, and the common allowance was half-a quart. The younger hands did
not begin with gin.
Q:'Was gin the first thing taken before any solid food was taken?--
A: Yes, and the breakfast was very light; those who took gin generally took only half-a
pint of tea and half a two penny loaf as breakfast.
Q:'When again was liquor brought in?-
A:At eleven o'clock. 'What was taken then?-Some took beer, some took gin again. In a
general way, they took a pint of porter at eleven o'clock. It was seldom the men took
more than the half-quart of gin.
Q: 'When again was liquor brought in ?
A:-At three o'clock, when some took beer and some gin, just the same as in the
morning. At five o'clock the beer and gin came in again, and was usually taken in the
same quantities. At seven o'clock the shop was closed.
Q:'What were the wages they received?-
A:Sixpence per hour, which, at the full work, made 6s. a-day, or 36s. a-week. Q:'Did
they make any reserves from this amount of wages ?
A:No; very few had anything for themselves at the end of the week.
Q:'How much of the habit of drinking was produced by the state of the workplace?
-A:I should say the greater part of it; because when men work by themselves, or only
two or three together, in cooler and less close places, there is scarcely any drinking
between times. Nearly all this drinking proceeds from the large shops, where the men
are crowded together in close rooms...
- From Sir Edwin Chadwick's official report on labor conditions- this excerpt is an interview
with a factory tailor (1842)
Document 4

Source: Political Cartoon on labor during the Industrial Revolution.


Document 5

Mr. -------- remarked that nothing could be so beneficial to a country as manufacture.


'You see these children, sir,' said he. 'In most parts of England poor children are a burden to their
parents and to the parish; here the parish, which would else have to support them, is rid of all
expense; they get their bread almost as soon as they can run about, and by the time they are
seven or eight years old bring in money. There is no idleness among us: they come at five in the
morning; we allow them half an hour for breakfast, and an hour for dinner; they leave work at
six, and another set relieves them for the night; the wheels never stand still.'
I was looking, while he spoke, at the unnatural dexterity with which the fingers of these
little creatures were playing in the machinery, half giddy myself with the noise and the endless
motion; and when he told me there was no rest in these walls, day or night, I thought that if
Dante had peopled one of his hells with children, here was a scene worthy to have supplied him
with new images of torment.
'These children then,' said I, 'have no time to receive instruction.' 'That, sir,' he replied 'is
the evil which we have found. Girls are employed here from the age you see them till they marry,
and then they know nothing about domestic work, not even how to mend a stocking or boil a
potato. But we are remedying this now, and send the children to school for an hour after they
have done work.' I asked if so much confinement did not injure their health. 'No' he replied, 'they
are as healthy as any children in the world could be. To be sure, many of them as they grew up
went off in consumptions, but consumption was the disease of the English. ...'
'We are well off for hands in Manchester,' said Mr. ------; 'manufacturers are favourable to
population, the poor are not afraid of having a family here, the parishes therefore have always
plenty to apprentice, and we take them as fast as they can supply us. In new manufacturing towns
they find it difficult to get a supply. Their only method is to send people round the country to get
children from their parents. Women usually undertake this business; they promise the parents to
provide for the children; one party is glad to be eased of a burden; and it answers well to the
other to find the young ones in food, lodging and clothes, and receive their wages.' 'But if these
children should be ill-used', said I. 'Sir,' he replied, 'it never can be the interest of the women to
use them ill, nor of the manufacturers to permit it.'
It would have been in vain to argue had I been disposed to it. Mr. ------- was a man of humane
and kindly nature, who would not himself use any thing cruelly, and judged of others by his own
feelings. I thought of the cities in Arabian romance, where all the inhabitants were enchanted:
here Commerce is the Queen witch, and I had no talisman strong enough to disenchant those
who were daily drinking of the golden cup of her charms.
-A skeptical Robert Southey interviews a proud factory owner. (1807)

Document 6
As to the conclusions I have come to from the working of my mill for 11 instead of 12 hours each
day, as previously, I am quite satisfied that both as much yarn and cloth may be produced at
quite as low a cost in 11 as in 12 hours. It is my intention to make a further reduction to 10½
hours, without the slightest fear of suffering loss. I find the hands work with greater energy and
spirit; they are more cheerful, and happy. All the arguments I have heard in favour of long time
appear based on an arithmetical question - if 11 produce so much, what will 12 or 15 hours
produce? This is correct, [for] the steam-engine, but try this on the horse, and you will find he
cannot compete with the engine, as he requires time to rest and feed.

There is more bad work made the last one or two hours of the day than the whole of the first
nine of ten hours. About 20 years ago, we had many orders for a style of goods. We had about
30 young women in our Manchester warehouse; I requested that they would work [instead of
11] 12 hours. At the end of the week, I found they had not a mere trifle more work done' but,
supposing there was some incidental cause for this, I requested they would work 13 hours the
following week, at the end of which they had produced less instead of more work. The
overlooker invited me to be in the room with them the last hour of the day. They were exhausted
and making bad work and little of it. I therefore reduced their time two hours, as before. Since
that time I have been an advocate for shorter hours of labour.
A Wise Factory Owner: Parliamentary Papers (1845)

Document 7
The history of all hitherto existing society is the history of class struggles... The modern
bourgeois society that has sprouted from the ruins of feudal society, has not done away with
class antagonisms...Society as a whole is more and more splitting up into two great hostile
camps, into two great classes directly facing each other bourgeoisie (factory owners) and
proletariat (factory workers /wage slaves). . .

The Communists disdain to conceal their views and aims. They openly declare that their ends
can be attained only by the forcible overthrow of all existing social conditions. Let the ruling
classes tremble at a Communistic revolution. The proletarians have nothing to lose but their
chains. They have a world to win. Working men of all countries, unite!

Karl Marx: Communist Manifesto 1848


SECTION II

Total Time- 1 hour, 30 minutes

Question 2 or Question 3

Suggested writing time: 35 minutes

Directions: Choose EITHER question 2 or question 3.


In your response you should do the following.

 Thesis: Present a thesis that makes a historically defensible claim and responds to all parts of
the question. The thesis must consist of one or more sentences located in one place, either in
the introduction or conclusion.
 Application of Historical Thinking Skills: Develop and support an argument that applies
historical thinking skills as directed by the question.
 Supporting the Arguments with Evidence: Utilizes specific examples of evidence to fully and
effectively substantiate the stated thesis or relevant argument.
 Synthesis: Extend the argument by explaining the connections between the argument and ONE
of the following.
 A development in a different historical period, situation, era, or geographical area.
 A course theme and/or approach to history that is not the focus of the essay (such as political,
economic, social, cultural, or intellectual history).
 A different discipline or field of inquiry (such as economics, government and politics, art history,
or anthropology).

3) Evaluate the extent to which the Chinese Revolution in 1949 marked a turning point in the triumph of
traditional Communism in world history, analyzing what changed and what stayed the same from the
period before the revolution to the period after it. (Historical Thinking Skill: Periodization)

or

2.) Evaluate the extent to which the achievement of Indian independence in 1947 was a significant
turning point in the in the world's history of decolonization.

In the development of your argument, explain what changed and what stayed the same from the period
immediately before the decolonization of India to the period immediately following it.. (Historical
Thinking Skill: Periodization)

STOP---END OF EXAM
AP
__________________________
Sample AP World History
2017 Multiple-Choice Answers
__________________________
Answers to Multiple-Choice Questions
1. (D) traditions and practices of Islam
2. (C) the rise and spread of Islam
3. (A) The development of the Silk Road
4. (A) Active encouragement of export and manufacturing for the world economy
5. (C) manufacture and export
6. (B) the environment in urban areas has become polluted and hazardous
7. (A) The convergence of world markets in the age of globalization
8. (C) patriarchy was viewed as legitimate
9. (A) The 19th Amendment to the U.S. Constitution, granting women the right to vote
10. (C) the failure by many liberals to grant women full equality and citizenship
11. (D) political and social acceptance of torture as a means of discipline and punishment for
suspected criminals
12. (D) The ideas of the Enlightenment
13. (A) The increase in the rights of the accused like innocent until proven guilty
14. (C) the Arab territories of Transjordan, Palestine, Syria, and Lebanon were divided among
Europeans
15. (D) a mandate system under European influence
16. (B) Berlin Conference
17. (B) 14 Points Plan
18. (A) lauds the possible benefits of the slave trade
19. (C) encourage the improved treatment of the slaves
20. (B) the success of the Haitian Revolution
21. (D) portraying the cities of Islam as the most virtuous places on Earth
22. (D) support the expansionist policies of the Umayyad Dynasty
23. (B) the philosophical and scholarly atmosphere of Islamic Spain
24. (A) an increase in the spread of U.S. military bases in places like Okinawa
25. (C) the use of military force as a means of promoting democracy
26. (D) Democracy was at risk, and should be protected at all costs
27. (D) The idea that the Pope and Catholic church were corrupt and in need of a religious
reformation
28. (A) the increasing sentiments that the Catholic church was interested in money more than
the gospel
29. (D) Calvinists
30. (B) the need to develop a uniform and public law code to unify his people
31. (A) The establishment of legal codes
32. (B) the relative equality of all citizens under the protection of the law in Mesopotamia
33. (A) Modern democracies are based on the rule of law
34. (A) diversifying diets and thereby triggering a population increase
35. (C) The population decline of Native Americans
36. (A) The rise of the Triangle Trade
37. (B) the expansionist policies of the first four caliphs and the Umayyad Dynasty
38. (A) the promotion of science, literature, and philosophy in Spain
39. (A) significance of understanding how to navigate the Monsoon winds
40. (C) Absolute monarchs
41. (A) the appeal of free and democratic elections in various countries
42. (C) the rise of fascism and dictatorship throughout Europe
43. (D) the cultural customs of the Persians under the Safavid Dynasty
44. (C) the Persians practiced Shi'a Islam, which venerated the lineage of Ali
45. (B) The proper role of government
46. (A) The Great Leap Forward and Cultural Revolution
47. (C) the popularity of other philosophies like Confucianism, and Buddhism among popular and
elite forces
48. (D) it was a turning point in the promotion of Buddhism
49. (B) the desire to put philosophical and political theory into practice
50. (A) Military documents from all of the major civilizations up to that point
51. (B) promote the ideals of the Chinese communist revolution
52. (B) mobilization of all social classes in support of the Cultural Revolution and it's omnipotent
leader
53. (C) China's liberalization of the economy after Deng Xiaoping opened the economy to
western business
54. (B) promote unity and integration in a post-Apartheid South Africa
55. (D) Mahandas Gandhi

AP
__________________________
Sample AP World History
2017 Free-Response Answers
__________________________
Question 1

This question is about similarities in migration patterns between Bantu


and American populations during the ancient era.

a) The biggest geographic similarity is that they both began in the northwest
before spreading south. The Americans began in Russia, then spread from
Alaska to the southern tip of Argentina- Cape Horn. The Bantus spread from
the Niger region to the southern tip of Africa- the cape of Good Hope.

The linguistic influence was similar in both cases. Bantu migrations


helped spread the various languages and dialects throughout Africa.
Similarly, the American migrations helped spread the indigenous languages
from north to south of the continent.

The agricultural impact was also shared in common. The Bantus


helped spread the technology of plowing and various cultivation techniques
throughout Africa. In a similar manner, the American migrations eventually
led to established civilizations like the Olmecs, Toltecs, Aztec, Inca, and
Mayas.

b) The main difference was their place of origins. The Americans originally
came from the Mongolia/ Russia area before they crossed the land bridge
across Alaska into the Americas. However, the Bantus began in Africa before
spreading throughout the rest of Africa. The Americans can be traced back
to the Mongoloid race whereas the Bantu are distinctly African.

c) The evidence of Clovis arrowheads is one difference resulting from the


migrations into the Americas. As the American nomads began chasing big
game like deer and buffalo across the continent, they developed arrowhead
technology to hunt animals. That accounts for the existence of native
American arrowheads scattered across the continent.

The Bantu societies never fully improved their agricultural methods. As a


result, mostly small scale farming with traditional technology and stone tools
were used across Africa until the time of imperialism.
Question 2

The question is based on comparing two passages on women and


citizenship during the age of the Roman empire.

a) The main difference between the two passages is that the first author
doesn't believe in women's equality whereas the second passage demands
full citizenship on equal terms. The first passage see women's equality as a
form of slavery over men, but the second passage views women's equality
as true freedom. Each author sees the other sex as the chaotic one causing
problems in society.

b)The first passage was written from a patriarchal perspective. The man
clearly expresses how he feels that women's equality means male slavery,
which is why he wants to dominate women in society.

The second passage expresses a belief in women's rights and equality.


Perhaps, she was influenced by democratic ideals. Maybe she was impacted
by the Roman democracy, which granted freedoms to the plebian (lower)
classes. She expresses an early fight for women's rights, that wouldn't really
be seen until the 19th century.
Question 3

This question asks you to explain why the Mongol conquest of Baghdad
in 1258 was a turning point in history.

a) The biggest effect of the Mongol conquest of Baghdad was the fall of the
Abassid Dynasty. The Abassids were a Muslim dynasty who ruled over the
Muslim world from 750-1258 C.E. They had been the largest and most
powerful empire in the region until they were crushed by the Mongols. The
Abassid caliph was murdered by the invaders as he was trampled by horses.
The fall of Baghdad led to a decline in literature and education in the city.
The city was a center for learning and scholarship before the Mongols. After
the Mongol invasion, the city was looted and books were burned. Much of
the knowledge had been lost due to the Mongol invasion. The next major
ruling Muslim empire would be ruled by the Ottoman Turks, no longer by the
Abassid Arabs.

b) One piece of evidence that contradicts the statements in a is that Islam


and Islamic empires still continued long after the fall of Baghdad. It's not like
the fall of Baghdad, led to the disappearance of Islam. On the contrary,
Islam spread and flourished under three very strong empires during the
modern era: Ottoman Turks, Persian Safavids, and Indian Mughals. You
might think that the fall of the greatest Islamic empire had a devastating
effect on the religion, but that wasn't the case. Islam continue to thrive long
after the fall of Baghdad. Plus, the holiest place in Islam will always be in
Mecca, regardless of where the caliph lives.

c) The Abassid caliph Harun al-Rashid ruled the empire at its high point. He
helped make the Abassids the greatest Islamic empire of its time. The
Abassids are known as the golden age of Islam. Harun al Rashid funded the
Dar al Hikma in Baghdad, or House of Knowledge. This was a scholarly
center where education and science flourished. Once the Mongols sacked the
city, all the knowledge and hard work of Harun Al Rashid had been laid to
waste.

Another possible development includes the rise of the Mongol empire


after Gengis Khan. After the Mongols came to power, they decided to spread
their empire as vast as they could extend it. Without the rise of the Mongols,
the conquest of Baghdad never would have been possible.

Question 4

This question is an excerpt from John Locke's Two Treatises of


Government.

a) One implication is that the people could overthrow their government in a


revolution and establish a democracy. Locke's ideas were actually put into
practice in all three democratic revolutions- American, French, and Haitian.
Locke said, the government can't rule without the "consent of the people" or
else they were justified in overthrowing the government and "may constitute
themselves a new legislature."

b) Most likely Locke was influenced by the Scientific Revolution. His ideas of
natural laws, and rights were influenced by the natural laws of science.
Locke just applied those scientific laws to politics, helping to found the field
of political science. He believed that people had inalienable rights and
nobody had the right "to harm another in his life, health, liberty, or
possessions." Scientists like Copernicus, Galileo, Bacon and Newton all
believed in the natural laws of the universe. Similarly, Locke believed in the
natural rights of citizens in a democracy.

c) The entire era of democratic revolutions was heavily influenced by Locke.


The revolutions in America, France, and Haiti were all influenced by Locke.
They all used his language in their founding political documents for their new
democracies. In America, Thomas Jefferson write about the rights of "life,
liberty, and the pursuit of happiness," heavily borrowed from Locke's
writings. In France, they established the rights for all French citizens based
on Locke's ideas. Haiti was founded on the principles that no one had the
right to take away their liberty or liberty.
SECTION II, PART A: DOCUMENT BASED QUESTION (EXPLAINED)

Document 1

This document is based on a chart of factory wages in a British city in


1830 C.E. The chart ranges from ages 11-61, and gives the brackets for
male and female wages. It clearly shows how at a young age females
actually made more money than males. However, after age 17 males were
paid higher wages than females. It shows how wages were not set at a
minimum wage as there were no government labor laws in 1830. It also
shows how kids and females were being used as factory labor. The chart
says a lot because it shows how worker's were not paid a uniform, objective
rate for the job, but one that was subjectively chosen by the employer. That
explains why women and kids were usually paid less than young men. It is
unclear if these factories hired a higher number of male or female workers.

Document 2

This is a fascinating document because it is testimony to the British


government from a woman miner during the industrial revolution. She
describes her difficult working conditions. She started working at 12 years
old. She worked a very difficult, and dirty job with long hours for very little
pay. She works a 12 hour shift every day with a one hour break for lunch. It
shows how difficult the working conditions were, and how women workers
were mistreated during the Industrial Revolution. At the end, she mentions
how her husband also abuses her at times. Definitely the working conditions
add stress on her family life as well, including the husband and children.

Document 3

In this classic government report from the industrial revolution era,


the author is reporting on the horrible working conditions. He interviews a
worker in this passage, and asks him about all of the alcohol he consumes
all day at work. The worker admits that they usually drank before work,
because they were depressed. The employer would provide the alcohol as
fuel to keep the worker's motivated. This document really demonstrates how
different working conditions were in those days in industrialized countries
today.

Document 4

Here, the political cartoon depicts a group of people pulling a big, fat
man smoking a cigar on a wagon whose caption reads "supported by child
labor." There are children and even girls pulling the wagon with the help of
an adult. The man, who represents a capitalist factory owner, is just sitting
there while the kids do all of the work. The cartoon is criticizing capitalism
and the factory bosses by saying that they are fat cats who do nothing but
smoke cigars and drink alcohol, living off the hard work of others- in this
case children. The cartoon would was probably written by a communist
sympathizer or at least a labor reform advocate.

Document 5

This document is a personal letter about an interview with a factory


owner explaining all of the benefits of child labor for not only himself but
also the children and their families. The interviewer is quite skeptical and
dubious about having children work in factories for such long hours under
such harsh conditions. The factory owner assures him that the kids benefit
immensely, because they make money for their families and receive some
education. The interviewer doesn't share the factory owners same
enthusiasm for child labor, but he doesn't publically express his discontent.
You really get the feel for how people justified child labor back in those days.

Document 6

In this reading from the Parliamentary Papers, the wise factory owner
explains his reasons for working his employees less hours. The factory owner
states that there is no reason to work the employees so many hours because
after about 11 hours, they stop being efficient or productive or the quality
suffers. He states, "there is more bad work made the last one or two hours
of the day than the whole of the first nine or ten hours." That was his
justification for reducing their work hours "without the slightest fear of
suffering loss" of profits. This perspective is unique because it is a capitalist
reason for reducing work hours, as opposed to a communist perspective
which would argue that all bosses are bad because they enforce the inhuman
capitalist system. This document, on the contrary, shows that there were
capitalist worker reformers in addition to the communists.

Document 7

This document is from the Communist Manifesto by Karl Marx and


Freidrich Engels- the most important document written during the industrial
revolution. The beginning of the document explains that the history of the
world has always been between two classes of society- rich and poor,
landlords and peasants. The only way for workers to get their fair share of
the profits is to overthrow the bosses in a violent revolution that leads to a
world controlled by the workers. The goals of communism were to unite all
workers from around the world. The final words of the document were a call
to action, "The proleterians have nothing to lose but their chains. They have
a world to win. Working men of all countries, unite!" Those are the words
that have influenced so many communist and socialist movements since
they were written in 1848. It was written right at the peak of
industrialization as a result of the mistreatment of workers like the others
you read about in this DBQ. This document is in direct opposition to the wise
factory owner document (6) because it says that there is no such thing as a
good capitalist. To them, a good capitalist (factory owner) is like a good
slave owner- meaningless. They're both horrible because the factory
systems and slavery were inherently evil; there was nothing any good slave
or factory owner could do to make it right since the workers weren't in
control of their own labor in either system. He refers to wage slavery as
worse than chattel slavery, because the workers were paid by the hours and
it didn't matter if they had food, clothing or shelter. However, at least in
slavery, the slave had food, and shelter to stay alive and work the next day.
Some communists believed that slaves were more valuable than hourly
wage workers or " wage-slaves", because slaves were taken care of, but
workers were discarded after their work hours were completed.
SECTION II, PART B: LONG ESSAY QUESTION (EXPLAINED)

Question 2

It is important to know some key facts and details for your essay
like...

(IDEAS & FACTS TO MENTION IN YOUR ESSAY)

 Before the revolution, there was a civil war, Nationalists v Communists

 Sun Yet Sen and then Chiang Kai Shek were the nationalist leaders.

 Mao Ze Dong was the Communist leader.

 During WWII, the forces united versus their common enemy, Japan.

 In 1949, the Communists came to power in China.

 Chinese communism was not traditional.

 Chinese communism was based on a peasant revolution, not workers.

 Mao wanted to apply the concepts of Marx to China.

 The Cultural Revolution spread across China.

 Revolutionaries promoted Communist propaganda all over.

 The Great Leap Forward was a failed program that led to famine.

 Communism in China is not viewed by outsiders as a success.

 The peasants remained mostly poor until globalization, after 1973.


 In 1973, Deng Xiaoping opened up China to Western businesses.

 Communism w a mix of capitalism continues today in China.

SECTION II, PART B: LONG ESSAY QUESTION (EXPLAINED)

Question 3

It is important to keep in mind that India was the "crown jewel" of the
British empire. When India gained independence, many other nations
thought that their dreams of liberation would soon come true. Were they
right? Let's look at a few key facts and concepts.

(IDEAS & FACTS TO MENTION IN YOUR ESSAY)

 Before 1947, Europeans held most of their colonial possessions.

 After 1947, India gained independence.

 India and Pakistan were both formed in 1947.

 After India, many other nations gained independence.

 India gained independence by using non-violent tactics.

 Algeria gained independence using violent resistance.

 Africa gained most of its independence in the 1960's.

 Ghana gained independence in the 1950's.

 Many nations gained independence after 1947.

 Most of the colonies didn't get their freedom until 20 years later.

 Vietnam was decolonized in the 1950's.

 After the French left Vietnam, the Americans invaded the country.
 In 1997, Hong Kong went back to China.

 The Treaty of Nanjing in 1847 gave Britain control of Hong Kong.

You might also like